Can You Get Something For Nothing?

"One creates from nothing. If you try to create from something you're just changing something. So in order to create something you first have to be able to create nothing." -Werner Erhard

One of the oldest adages in existence is you can't get something for nothing, as over a million websites will tell you, including not-so-subtly, cartoonstock.

And, most often when people bring this up to me, it's in an attempt to prove the existence of God -- and the insufficiency of the Big Bang -- by pointing to the Universe.

Image credit: chaospet.

Well, let's take this question as seriously as our knowledge allows us to. (And by that, I mean physically, rather than philosophically or theologically.) In physics, can you get something for nothing? And if so, what can you and can't you get?

In many ways, yes, you can. In fact, in many ways, getting something when you have nothing is unavoidable! (Although you can't necessarily get anything you want.)

For example, take a box and empty it, so that all you've got is some totally empty space, like above. An ideal, perfect, empty vacuum. Now, what's in that box?

Did you guess nothing? Well, it turns out that empty space isn't so empty.

One of the consequences of Heisenberg's Uncertainty Principle -- that you can't know a quantum state's energy exactly for a finite duration of time -- means that when you're talking about very short time intervals, there are large uncertainties in the energy of a system. Over short enough timescales, the energies are large enough that particle-antiparticle pairs wink in-and-out of existence all the time!

"That's crazy talk," you say. Prove it!

And they did.

Take two identical, uncharged, parallel metal plates, and put them close to one another. The vacuum fluctuations in between the plates cause there to be a pressure pushing the plates together. This isn't the gravitational force or an electromagnetic force, but a force due to empty space itself.

This experiment -- first done in 1948 but repeated many times (under many conditions) -- was a rousing success, and has many immediate, far-reaching and fantastic consequences.

Image credit: thinkquest.

Black holes decay!

The space near a black hole is, of course, filled with these particle-antiparticle pairs, just like space everywhere else. But create a pair close to the event horizon, and one of the two can fall in! The other one, being outside the event horizon, can escape, carrying energy away, and becoming real. These particles that escape are known as Hawking radiation.

They provide the seeds for all the structure in our Universe!

When the Universe inflates, or expands exponentially (before the Big Bang), these quantum fluctuations also expand, and get stretched across the Universe faster than they can annihilate one another. These fluctuations show up as regions with slightly more (for positive fluctuations) or less (for negative ones) energy, which then grow into structure (like clusters, galaxies, and stars) and voids as the Universe ages.

Image credit: CLEF-SSH.

And if you start with enough energy, you can take all of the real matter and antimatter pairs that exist, and create more matter than antimatter, giving us a Universe where we have something, today, rather than nothing.

Now, that's what we know we can get, even from nothing. But there are many things we can't do, either practically or theoretically: violate charge or energy conservation, decrease the total entropy of the Universe, or figure out where our initially inflating Universe came from. (Yet!) But we definitely can get something for nothing; quantum field theory not only allows it, it demands it. But it remains to be seen whether we can get everything for nothing. If we ever figure it out, I'll make sure you're among the first to know!

More like this

The other one, being outside the event horizon, can escape, carrying energy away, and becoming real.

And here, I was totally expecting that to be a link to this recent XKCD. Sure fooled me!

By Randy Owens (not verified) on 02 Feb 2011 #permalink

Is there any reason why the antimatter particle of the pair should form preferentially on the near side to the event horizon? If not, then wouldn't matter and antimatter be added at roughly equal rates both to the black hole and to the rest of the universe? Wouldn't they be more or less stable systems?

The obvious difference between the experiment showing the Casimir effect and the universe at the time of the Big Bang is that those metal plates are in an existing space-time framework. In what sort of a space-time framework did the Big Bang take place, and how did that come to exist?

By Scott Brickner (not verified) on 02 Feb 2011 #permalink

Kind of like Scott's asking, the described effect requires "space" to initiate which is technically not the same thing as "nothing" since "space" is and has energy while "nothing" is not and has not. A supposed pre-universe condition of nothing at all has no space and perhaps not even a singularity (which is something). "Nothing" has no energy and by definition does not exist at all. Perhaps this is why our current physics become as yet insufficient at "zero".
Call this comment a something for nothing if you wish.

By Lloyd Hargrove (not verified) on 02 Feb 2011 #permalink

Ethan:

Getting something from nothing makes me think about the cold fusion experiments by Fleishmann and Pons. How they claimed to get more energy out then put in along with an excess of tritium. I would love it if you would do a write up on the topic. I am wondering what you think about the experiment. After all it kinda goes against everything we think we know about physics and the Universe we live in.

@2: sure, Hawking radiation would be a mix of matter and antimatter, but both would carry energy away from the black hole and cause it to evaporate. Antimatter doesn't have negative energy.

OK, still don't get this. If both matter and antimatter particles have mass then why doesn't the black hole expand rather than dissolve?

Why doesn't Hawking radiation lead to the universe having one extra particle and the black hole having one extra particle?

By Yossarian (not verified) on 03 Feb 2011 #permalink

@6. But doesn't each antimatter particle that radiates away from the black hole have, on average, a corresponding matter particle radiating away elsewhere. Whether they annihilate together or separately the net energy when they do annihilate (on each side of the event horizon) is zero, isn't it?

your usual level of fantastic. thanks for a great post.

@Yossarian: Remember that mass is a form of energy: E = mc^2. So if one of the virtual particles escapes and becomes a real particle, that much energy is lost to the black hole. The process is not that important for stellar mass black holes: it would take several orders of magnitude longer than the lifetime of the universe for such a black hole to dissolve by Hawking radiation. However, certain cosmological theories over the years have predicted the existence of microscopic black holes, for which Hawking radiation is a significant effect.

@Kym: Although the particles being radiated away from the black hole are a mixture of matter and antimatter, they are also being radiated in different directions at different times, so there is no guarantee that the escaping particles will annihilate each other (remember, they have to have an initial radial velocity close to c to get out). Even if they do, the resulting photons will generally not be directed toward the black hole (momentum as well as energy must be conserved; yes, photons have momentum, but that's a different post).

By Eric Lund (not verified) on 03 Feb 2011 #permalink

I was totally thinking the same thing as #7/8. I'm certainly no physicist, but the black hole swallowing one of the pair of spontaneously created particle/anti-particle should add mass to the black hole, right? Right? I know much of this stuff is counter-intuitive by nature, but this seems... I don't know. Entanglement and all the other weird quantum things I've ever heard of are weird enough, but I can somehow accept them. That added mass = dissipation, is infinitely more perplexing to me.

By cgauthier (not verified) on 03 Feb 2011 #permalink

"That added mass = dissipation, is infinitely more perplexing to me."

The black hole didn't gain mass, a particle escaped -- antimatter is just matter traveling backward in time. You see it as falling in, but the particle sees itself escaping. :)

(Don't ask me what happens when normal half of the pair falls in and the anti escapes -- I don't have a smart-ass answer for that one...yet).

The problem with QM is epistemological â our standard language describes events common to our experience; itâs entirely inadequate to explain things beyond our experience like QM or heaven or anything else outside The Matrix we live inâ¦

To be fair, I notice you link to Wikipedia, but, again, non-physicist here. Numbers, letters and squiggly lines, arranged in some esoteric code, are no explanation for the layman.

The article does mention something about the black hole losing energy when the escaping particle/anti-particle escapes, but I don't see how that counteracts the addition of the other particle/anti-particle's mass.

And if it happens to be the anti-particle which is taken into the black hole, the energy from the annihilation w/ the black hole's matter should still remain in the black hole, right? And either way, particles and anti-particles should be entering and escaping the hole at proportions that are roughly 50/50, right? I mean, it's either heads or tails.

I don't know. I just don't think the understanding of Hawking Radiation should be taken for granted. And I don't think the Wikipedia article does a great job at explaining it to the laymen*.

*Or, I could be much denser than your typical laymen. Perhaps so dense, that, as virtual particle/anti-particle pairs are separated at my event horizon and I absorb one of each said pair, I lose mass and will eventually dissipate.

By cgauthier (not verified) on 03 Feb 2011 #permalink

The black hole didn't gain mass, a particle escaped -- antimatter is just matter traveling backward in time.

I've never, ever, ever ever heard that explanation for antimatter ever ever. Sorry, and not to call you a crank or liar, but I need a bit more confirmation on that idea. Wikipedia says nothing about antimatter being "matter traveling backward in time".

When matter/antimatter annihilate, are the gamma rays created travelling forward, or backward in time? Or both? That sounds even more ridiculous than what I was originally questioning.

Any second opinions?

By cgauthier (not verified) on 03 Feb 2011 #permalink

Also, Doug, I know that language is a poor substitute for the math, regarding QM, but I'm pretty sure language is all there is to descibe Heaven. You know, seeing as how QM has math and real experimental results and heaven was invented as an idea, using language, and has no phenomena to see or math to measure it by.

By cgauthier (not verified) on 03 Feb 2011 #permalink

My understanding of how the black hole is dissipating energy is that the energy to create the particle/antiparticle pair comes from the black hole's gravitational energy. Normally they'd annihilate and return the energy but when one is captured by the black hole and the other escapes, the black hole only gets half the energy back.

This is probably a grossly simplified explanation as I'm not a physicist.

If I understand this, mathematically, a positron can be modeled as an electron moving backwards in time, and the equations work out. Richard Feynman worked on this. At one point, he suggested that all the matter in the universe might be a single electron moving backward and forward; later he said he had proven to his own satisfaction that there had to be at least two particles.

However, I am not a physicist, and I hope one of the physicists here will give more detail.

Meta: Wikipedia can be a good starting place, but it's far from complete (even if it aspires to that). The absence of something from Wikipedia doesn't mean it's invalid.

Since the Casimir force arises from the exclusion of electromagnetic modes from the space in between the conducting plates (they have to be conducting), I think it's fair to say that the Casimir force is an electromagnetic force.

Great comments and further inquiry. I am in the camp of using the ideas, metaphors and the thinking process of physicist to help regular people in the real world make better decisions. The big point I take away from this is that is never a state of "nothing."

Energy exist everywhere in the universe (even down to the strings of string theory) and so something is always possible to create by tranforming the energy that is always there. This has parallels in life from the inventiveness of the entrepreneur to "reinventing" ourselves when it appears all was lost.

Scientist, like Brian Greene, who was recently in our area are also advocating that science become more a part of our everyday lives in improving the quality of our thinking.

My book, The Art of Quantum Planning, attempts to break this ground for businesses and organizations. We can't all be physicist, but we can think like them a lot more than we
give ourselves credit for. This kind of thinking will help us continue to create a new and better world, right here on Earth.

Of course, you can't get more out of a black hole via Hawking radiation than fell into it in the first place (the black hole evaporates as soon as the two quantities become equal) so it's not really something for nothing is it? More like a capacitor or a battery.

By Ian Kemmish (not verified) on 03 Feb 2011 #permalink

Great!. The pictures are excellent; I teach physics in HS, and I can use the pix with the story very nicely when I get to QM. I already tell them about virtual particles and Casimir effect, but I always need better illustrations to go with the talk and better talk to go with the pix (I'll cite your site, when I get there)

By Gary Allan (not verified) on 03 Feb 2011 #permalink

About the more general particle/anti-particle formation, does that require energy to be present, or can it happen in a case with no background energy? I originally thought that it required energy, but the bit you mentioned about Heisenberg seems to imply that it can happen even without energy (since even if there is no energy on average the uncertainty means there still can be some locally). Is that correct or am I missing something?

By TheBlackCat (not verified) on 03 Feb 2011 #permalink

When the Universe inflates, or expands exponentially (before the Big Bang)...

Uh, wut?

Last I heard, "before the Big Bang" is either nonsense (time itself being one product of the BB) or unknowable (BB considered as a cosmic Reset erasing all prior information).

And is there, as another science blogger recently stated, something in common between vacuum energy and "dark" energy causing the universe to expand?

By Pierce R. Butler (not verified) on 03 Feb 2011 #permalink

It is very simple, nothing interacting with nothing produces nothing, not something. It is impossible for something to come from nothing. That is the law of logic. Defy logic and you have entered never never land.

By Chuck Nance (not verified) on 03 Feb 2011 #permalink

It is not possible to produce something from nothing. Based on logic is won't work. Nothing interacting with nothing equals nothing.

By Chuck Nance (not verified) on 03 Feb 2011 #permalink

Hey Chuck, did you miss the part where the Casimir effect was demonstrated experimentally? The real, actual world really, actually works this way.

In the face of disconfirming evidence, don't you think you should reassess your position?

It is very simple, nothing interacting with nothing produces nothing, not something. It is impossible for something to come from nothing. That is the law of logic. Defy logic and you have entered never never land.

The problem is that you are conflating two conceptually different things: "Nothing at all -- no space, time, energy, or physical laws", and "a vacuum in which the laws of quantum mechanics operate".

Or in other words, you are committing the logical fallacy of equivocation. You may be doing so inadvertently, but you are doing it nonetheless. Science has both evidence and correct logic.

In order to make your argument work, you have to show that there really was "nothing at all" before the big bang -- no space, time, energy, or physical laws whatsoever.

Good luck with that!

By Owlmirror (not verified) on 03 Feb 2011 #permalink

"Hey Chuck, did you miss the part where the Casimir effect was demonstrated experimentally? The real, actual world really, actually works this way.

In the face of disconfirming evidence, don't you think you should reassess your position?"

I think physicians should be more careful in their explanations. This whole idea that something can come from nothing doesn't just defy god, but it defies science. Something coming from nothing should be an observation we're still trying to untangle and explain. It's a fact that it seems as though something comes from nothing, but if you have any appreciation for logic and reason, you should open your mind to a possible explanation. I don't see why hidden(from us) dimensions can't account for the possible interaction that leads to the "spontaneous" creation of antimatter/matter particles. Doesn't dark matter alone open you up to a possible explanation for the effect in question? I know it seems far fetched, but isn't it more far fetched to say something comes from nothing? I hate when people try to draw attention to science with counterintuitive facts about reality, because those aren't realities...they're the next deep questions we need to answer. It's great when the idea that something comes from nothing is presented as a question to be answered...which implies it isn't really nothing. Something comes from something. Open your mind up to an explanation, otherwise you chose faith over wonder.

John Baez explains a little more about Hawking radiation here, and says that the usual particle/antiparticle explanation is not so great, or at least not obviously connected to the actual theory. To summarize (if I've understood it correctly) the curvature of spacetime near the black hole means that the lowest energy state near the hole is different than the lowest energy state far away -- what we normally call "vacuum". From far away the area near the black hole appears to be at a higher energy state, and is thus emitting energy.

Hey the Casimir Force!
I've got a question about that one. I was reading about this one crazy magnetar thing and it said that magnetars have such strong magnetic fields that they polarize the vacuum...huh? Is that related to the Casimir Force? I mean, since the plates have to be conductive, I thought it might be what the article was referring to, somehow. Is it?

Non-physicist here. Do vacuum fluctuations have what might be described as a wavelength? Does the distance between the two plates matter and could they be pushed together faster than gravity could account for?

It's misleading to conclude that we get something from nothing when the conclusion is based on a misconception about "empty" space being nothing.

A vaccuum is in reality never empty but necessarily has a minimum level of energy, which is what is manifesting as virtual particle pairs. So we're not getting something from nothing but something from something.

By troels.jakobsen (not verified) on 03 Feb 2011 #permalink

It is very simple, nothing interacting with nothing produces nothing, not something. It is impossible for something to come from nothing. That is the law of logic. Defy logic and you have entered never never land.

"Law of logic?" What the hell? Are we talking about inference rules for grammatical statements, or are we talking about the ontology of the universe here? Because "laws of logic" refer to inference rules, not stuff about it being "impossible for something to come from nothing."

Or are you suggesting that your claim that it is impossible for something to come from nothing is some sort of tautology (based perhaps on simply defining nothing to mean "that from which something cannot come")? If so, hopefully you understand why arguments from tautology are vacuous -- a tautology is the nothing from which no inference of substance comes.

Thanks Ethan just the answer I needed. Are we any closer to understanding the state of affairs before the planck time? Do you think string theory or loop quantum gravity have any future as explanations?

By Shane Norman (not verified) on 03 Feb 2011 #permalink

Frankly, I'm not convinced that the vacuum has particles popping in and out of existance. Even in the Casimir effect, you have to introduce plates or something else to even measure it. How's that for a vacuum? Come on!

By Raskolnikov (not verified) on 03 Feb 2011 #permalink

about hawking radiation :

It makes no difference if the matter particle escapes and the ant-matter particle falls in or otherwise. The black hole loses energy.
Imagine 2 electron-positron( anti-electron) pairs being created through quantum fluctuations at the edge of the event horizon. In one pair the electron falls into the black hole and the positron escapes.
In the other pair it's the reverse. The net result is that now there is an extra electron and a positron with each a mass of about 0.5 MeV/c² (antiparticles don't have anti-mass). Schould those two particles ever meet eachother they will anihilate producing a pair of gamma rays with a combined energy of about 1MeV. Conservation of energy tells you this energy has to come from somewhere : the black hole.
Since mass and energy are equivalent the black hole lost mass.

By Frank Van Bragt (not verified) on 03 Feb 2011 #permalink

There is an alternative explanation to the Hawkins radiation, seen as quantum tunneling effect through the event horizon of the black hole. As a particle approaches the e.h. from the "inside" the possibility of that particle being in the "outside" increases. The particle-antiparticle explanation is just complementary to this.

I confess this struck me as amusing:
@14

To be fair, I notice you link to Wikipedia, but, again, non-physicist here. Numbers, letters and squiggly lines, arranged in some esoteric code, are no explanation for the layman.

Learning is like a baby drinking milk; it requires at least a tiny bit of participation on the part of the baby.

It only takes a few seconds of time to notice that the section linked carries the heading "Calculations" and is by necessity likely to contain the mathematical details. Another second or two of scrolling down the page would have led you to the section titled "Analogies" which includes the math-free, layman-friendly bits you were (ostensibly) looking for.

Mathematics is the natural language of our universe, and it's quite often not hubris, arrogance, or an attempt to obfuscate when physicists use it while trying to explain physical concepts. In fact, almost anyone willing to try can actually learn quite a lot of the math basics from wikipedia by clicking on some of the words they aren't familiar with until they reach their comfort level, and working back up from there. You might even find the experience nourishing!

As we continue down the statistical side of physics, I continue to have a nagging question that rattles around in the back of my head.

Are we really sure that this stuff is random? In the Casimir plate described above, we can measure some force between the plates. But the plates are isolated to the best of our ability and understanding of that isolation. Are we certain that the plates are isolated from all possible outside effects?

A few hundred years ago, noone understood light outside of the wavelengths we can see. It seemed like magic (or randomness) that some animals could see things when there was no light (that we could see), or easily see things that looked to be near-perfect camouflage to us.

I understand we are able to use the principles of quantum mechanics to do work today, but ancient peoples were able to figure out how to make bronze and other combined metals without understanding the atom too.

Root of the question is this: Are things at the quantum level truly as random as we think, or are there additional forces at work that we have yet to perceive and account for?

Marc A: Something coming from nothing should be an observation we're still trying to untangle and explain.

Dr. Siegel DID explain it to you. The universe does not make a distinction between "nothing" and "equal and opposite virtual particles." The two states are equivalent, and on a the scale of the Heisenberg uncertainty principle a "piece" of the universe can flip back and forth between the nothnig state and the something state. Get it out of your head right now that the nothing state is the baseline and the something state is a deviation from this baseline; they are quantum mechanically equivalent.

The Casimir plates do not create this effect, they simply measure the pressure from it. Take away the plates and it still happens: you just have no way of measuring it any more.

The bit about QM demanding that we get something from nothing is obvious once you grok that the two states (vacuum, equal and opposite virtual partices) are equivalent. For a microscopic space to remain "nothing" for all time and never produce virtual particles would be like flipping a (fair) coin every nanosecond for the entire age of the universe and always getting heads. That ain't going to happen.

Now for something completely different...

crd2 @5: Getting something from nothing makes me think about the cold fusion experiments by Fleishmann and Pons...After all it kinda goes against everything we think we know about physics and the Universe we live in.

Pons and Fleishman claimed to have produced fusion. Fusion (and fission) does not create something from nothing, it merely converts mass into energy. The sun does that all the time, as do physicists using particle accelerators.

What makes the P&F claim extroadinary was that they claimed to have produced fusion using a benchtop electrochemical setup, and without the emission of gamma rays. The first claim was extroadinary from an engineering perspective, while the second contradicts the quite extensive understanding we have of tritium fusion. Both claims proved to be irreproducible.

"Dr. Siegel DID explain it to you. The universe does not make a distinction between "nothing" and "equal and opposite virtual particles." The two states are equivalent, and on a the scale of the Heisenberg uncertainty principle a "piece" of the universe can flip back and forth between the nothnig state and the something state. Get it out of your head right now that the nothing state is the baseline and the something state is a deviation from this baseline; they are quantum mechanically equivalent.

The Casimir plates do not create this effect, they simply measure the pressure from it. Take away the plates and it still happens: you just have no way of measuring it any more.

The bit about QM demanding that we get something from nothing is obvious once you grok that the two states (vacuum, equal and opposite virtual partices) are equivalent. For a microscopic space to remain "nothing" for all time and never produce virtual particles would be like flipping a (fair) coin every nanosecond for the entire age of the universe and always getting heads. That ain't going to happen."

I'm afraid you're not getting the point. I understand how these two things can be considered one in the same in the framework of a theory, but the fact that you're able to distinguish the two things you say are equivalent makes them unequivalent in "some regard". I know that's vague, but I think I'm being intellectually honest. By the way, the uncertainty principle is based on reality, not the other way around.

The reason mass is only lost to a blackhole in hawking radiation, never gained (for the quantum mechanics version of never that has some infinitesimal caveat) is that above the limits of heisenberg uncertainty the conservation laws apply. A photon is welcome to turn into a shower of electrons, but it's going to have to collapse back into the equivalent of a photon pretty fast. If a virtual particle is pulled into some neighboring system (in this case, a black hole), then all of its virtual partners are going to have to do something to keep the ledger balanced: this may mean reforming into a particle with less energy than they began with, or "stealing" energy from some other system. It's like musical chairs: while the music is playing the particles dance around, but once the time is up, you've got to find a seat.

Normally when a particle and an antiparticle meet, you get a flash of energy. This doesn't happen in the vacuum, because the energy produced is just offsetting the energy used to create the pair. So when the blackhole pulls one of a vacuum pair in, the pair has a fixed amount of time to account for the energy that created them. The easiest way for this to happen is for one of the pair to find an instance of its antiparticle to collide with. Again, normally this would create a tiny explosion of energy and no mass would be lost, but because this interaction is "balancing the ledger", it occurs with a blip, some particle inside the blackhole no longer exists, and the previously virtual particle that escaped from the black hole has enough energy to continue existing.

Now, either particle is welcome to sacrifice itself for the other. But there particle escaping is heading out towards empty space, and the particle being pulled into the black hole is headed at high speed for a very dense grouping of matter. So the one inside the blackhole is far more likely to find a partner, so hawking radiation will tend to leak from a blackhole for thermodynamic type reasons.

If an action can't be balanced within the limits set by Heisenberg, it just doesn't occur. So if you had a blackhole full of only antimatter, more antimatter would never fall in. Virtual particle pairs on the edge would always fall matter end in, or else re-annihilate. It's a bit odd that the actions of the particle are dictated by events in the future, that you can pay the energy cost after the event has occurred, but that's QM for you.

Marc A: I'm afraid you're not getting the point.

I think you got my goat up with your "something from nothing defies science" comment in @30. This is not the case. It does not defy sience at all; it is perfectly explained by scientific theory.

QM without extra dimensions is perfectly adequate for explaining what's going on. You are right that the two states are detectably different, but this does not prevent the flip from happening. And in fact QM allows all sorts of other equivalent-but-detectably-different flips, too. A >1.022 MeV xray can spontaneously convert into a positron and electron. Solar neutrinos can oscillate (i.e., flip) their flavor. Weird? Yes. Counterintuitive? Yes. Unscientific and defying explanation? Not at all.

I understand how these two things can be considered one in the same in the framework of a theory, but the fact that you're able to distinguish the two things you say are equivalent makes them unequivalent in "some regard"

We are able to distinguish them because we try to impose our own macroscopic views on a system they don't apply to. You are conflating how we conceptualize phenomena that is utterly alien to our experiences with how the phenomena actually is.

By TheBlackCat (not verified) on 04 Feb 2011 #permalink

As Hobbes would say, the "something can't come from nothing" is an arbitrary decision. Until we have proof of this statement it cannot be taken as fact.
The real problem with this Casimir effect is that it takes vacuum (zero energy) and converts it into energy. When anti-matter and matter collide they create an explosion which releases energy. This would violate the conservation of energy and matter as "empty space" would quickly fill up with excess energy from the explosions.
So this effect must be understood in a different way. For instance, why does it happen between two plates? Is it possible that the Casimir effect is due to the uncertainty principle acting on the matter within the plates?
The other option is that we discover the law of conservation of matter and energy is wrong. This true is an arbitrary definition rather than a statement of empirically proven reality. It is possible that Newton was wrong about this, in the same way that he was wrong about the universality of stable space-time.
Of course if the law is incorrect, and the big bang was created from this effect, then each point of "space" in the universe is a potential new big bang waiting to happen (which would likely be very dangerous). This doesn't make the theory (that the law is incorrect) false, but it does question it.

@ Vicki:

mathematically, a positron can be modeled as an electron moving backwards in time, and the equations work out.

AFAIU some of the early particle descriptions (Dirac's equations) could be interpreted such. Not generally, no, since CP violation and what not appears in actual physics.

@ Pierce R. Butler:

Last I heard, "before the Big Bang" is either nonsense (time itself being one product of the BB) or unknowable (BB considered as a cosmic Reset erasing all prior information).

If you follow the link labeled "inflation", Ethan walks you through all that, and IIRC the vacuum/dark energy relation (or look it up in Wikipedia, it's there IIRC), in a series of posts.

Money quote:

"But when inflation came along, all of that changed. No longer could we extrapolate all the way back to a singularity. If we wound the clock of the Universe backwards, we would discover something remarkable. At some point, about 10-30 seconds before we would anticipate running into that singularity, the Universe instead would undergo inflation (in reverse, if we're looking backwards), and we have no evidence for anything that came before it.

The Big Bang, instead of being a singularity, is the set of initial conditions of an extremely hot, dense, expanding Universe that exists immediately after the end of inflation."

By Torbjörn Lars… (not verified) on 04 Feb 2011 #permalink

Are things at the quantum level truly as random as we think, or are there additional forces at work that we have yet to perceive and account for?

The answer is that things at the quantum level are entirely random. If there were any "forces at work" beyond statistical randomness, recognized or not, hypothesized or not, the results would be non-random. Unknown forces would produce unknown results, not random results. "Random" does not mean some particular pattern we don't recognize or understand. Mathematically, it means no pattern at all. And the results of empirical experiments confirm that it is random, not simply "unpredicted" but "unpredictable".

Such is my understanding.

@ Thomas M:

they polarize the vacuum...huh? Is that related to the Casimir Force?

Indirectly.

Vacuum polarization in QED is virtual particle pairs that the field creates and annihilates, which shows up in the theory and is AFAU called self energy of the photon. (Disclaimer: have not studied quantum field theory.)

Vacuum energy is the zero point (lowest state) energy of all particle fields self energy (so not only photons but all other bosons _and_ fermions).

@ Steve:

Do vacuum fluctuations have what might be described as a wavelength?

Yes, those fluctuations are the virtual particle-antiparticle pairs, and particles have all sorts of wavelengths associated to them.

The distance of the plates matter which can be realized by asking what happens if you separate the plates to infinity (no effect). I'm sorry but the gravitation question makes no sense to me, this is a non-gravitational effect.

By Torbjörn Lars… (not verified) on 04 Feb 2011 #permalink

The real problem with this Casimir effect is that it takes vacuum (zero energy) and converts it into energy.

that's not a problem, that's a solution. the problem is the notion that vacuum has zero energy, because zero is a certainty and Heisenberg rules certainties out. randomly created then rapidly uncreated energy blips fuzzies the edges of that zero so that it'll fit into the uncertainties demanded by quantum mech; this solves a problem.

it only seems to create a problem because we're trying to grok a quantum-scale phenomenon with macroscale intuitions. that doesn't work. the very small just doesn't make that kind of sense, and trying to make it so will only give you a headache while misleading you to the wrong conclusions.

By Nomen Nescio (not verified) on 04 Feb 2011 #permalink

@ Raskolnikov:

I'm not convinced that the vacuum has particles popping in and out of existance. ... How's that for a vacuum?

That is what is observed, and so that is how we nowadays understand vacuum, filled with fields and so particles. How else could it be? :-D

@ unbound, tmaxPA:

Are things at the quantum level truly as random as we think, or are there additional forces at work that we have yet to perceive and account for?

"Random" is meaningless without context, I believe you mean stochastic.

But quantum mechanics is deterministic as well (takes states deterministically to states) so truly stochastic but not "truly random" or "unpredictable".

Quantum mechanics denies hidden variables, and besides testing for a LUCA in biology this is among the best tested facts we have. (Bell test experiments at something like 10^-20 uncertainty; I haven't checked though. Universal common ancestor test is ~ 10^-2000 uncertainty ... Evolution rocks!)

By Torbjörn Lars… (not verified) on 04 Feb 2011 #permalink

When you get life from non-life, then we'll talk, Ethan. M'kay?

By Laughing Man (not verified) on 04 Feb 2011 #permalink

Delightful!
And since I think I understand it I presume I don't?

By Jonn Mero (not verified) on 04 Feb 2011 #permalink

Is the Werner Erhard you cite at the beginning of the article the same Werner Erhard who created EST? And who said this about L. Ron Hubbard: "I have a lot of respect for L. Ron Hubbard and I consider him to be a genius and perhaps less acknowledged than he ought to be?" If so, what strange bedfellows you keep.

@ Marc A:

This whole idea that something can come from nothing doesn't just defy god, but it defies science.

You have got that backwards. We currently observe something, and the idea that when we go back in time we suddenly "get nothing from something" is _the_ counter-intuitive but at times correct statement. But as Siegel explains here, or Vic Stenger in his books, those states are the same type of physics, just observationally different, so no problem.

Oh, and that "defy god" doesn't happen, since we have no evidence for gods.

the fact that you're able to distinguish the two things you say are equivalent makes them unequivalent in "some regard".

This is trivial but non-consequential here. Colors can be distinguished by wavelengths so they are nonequivalent (not the same). But equivalently they are equivalent (photons with wavelengths).

@ Chuck Nance, Marc A:

Defy logic and you have entered never never land.

counterintuitive facts about reality, because those aren't realities...

You are both wrong. Science has nothing in common with common sense and philosophical logic.

Common sense is formed by evolution and observation of some phenomena at mesoscales, while science handles all phenomena at all scales (in principle). The first thing you learn at university, at least in physics, is to dump the old bad mindset and learn actual physics. (Well, ideally, some never learn...)

Philosophical-religious logic (aka "truth", or worse, "Truth", in the vernacular) is contingent on context (say, either plane geometry or hyperbolic geometry, either christian gods or hindu gods), while results of science method (facts and theory) are observed to be universal.

Facts and theories, while having quantifiable uncertainty, beats common sense and/or truths hands down.

By Torbjörn Lars… (not verified) on 04 Feb 2011 #permalink

Billy Preston can teach you all you need to know about nothing from nothing.

@ Laughing Man:

When you get life from non-life, then we'll talk,

Ludicrous, because a) this is spontaneous generation not spontaneous creation, so biology not physics b) spontaneous generation was rejected hundreds of years ago, replaced by evolution at the time of Darwin and Wallace.

Not that it pertain to the subject, m'kay. But we currently know _too much_ about chemical evolution and how it gets systems to biological evolution. The number of potential pathways is too large to zero in on any one specific, it is an embarrassment of riches akin to the recent Kepler results pointing to many paths to planetary systems.

[In fact, I'm looking forward to the constraining of chemical evolution pathways that will take place when we first get to know habitable exoplanets and their environments, later no doubt now (since we found other habitable planets, yay!) inhabited planets and _their_ environments. If we are lucky we can zoom in to how it happened on Earth. If not we will get to know the large number of ways life gets started.]

By Torbjörn Lars… (not verified) on 04 Feb 2011 #permalink

@LaughingMan...funny...every time I talk to theists about abiogenesis, they move the goalposts to "you can't get something from nothing."

Take your goal post shifting and place it where the sun don't shine.

Chemistry. Life from non-life is chemistry. That's ALL it is. A chemical reaction that took place about 300 million years after the Earth formed. You don't need scientists with beakers to have chemical reactions, you know. They happen all the time. Organic AND inorganic. In nature.

Just because we haven't teased out the precise equation, that does not then allow you to insert "god did it" there.

And please don't start about "odds". The odds of life arising from non-life on Earth is 100%. A dead-shot certainty. Because we are evidence that it happened.

Just chemistry. Nothing more. Nothing less.

The odds of life arising from non-life on Earth is 100%. A dead-shot certainty. Because we are evidence that it happened.

The Bible is the Word of God. It says so in the Bible, so it's 100% true. (Same argument, different topic.)

So tell me, why isn't it that we are the evidence that alien life seeded this planet as many evolutionists claim?

By Laughing Man (not verified) on 04 Feb 2011 #permalink

BTW, if "we're the evidence" that abiogenesis happened, could you please recreate that in a lab for me? Thanks. I'd appreciate it.

By Laughing Man (not verified) on 04 Feb 2011 #permalink

So tell me, why isn't it that we are the evidence that alien life seeded this planet as many evolutionists claim?

shuffling abiogenesis off onto some other planet doesn't make abiogenesis not happen, y'know. (and who are these "many" who hew to panspermia, anyway? i'd like to ask them for their evidence. seems to me that unless they have some, the notion is subject to Occam's razor.)

By Nomen Nescio (not verified) on 04 Feb 2011 #permalink

"If I understand this, mathematically, a positron can be modeled as an electron moving backwards in time, and the equations work out."

It makes more sense on the Feynman diagram (where time direction arrows are reversed for anti-matter), for example:
http://en.wikipedia.org/wiki/File:Beta_Negative_Decay.svg

i'd like to ask them for their evidence.

Their evidence is as follows:

*ahem*

"we are evidence that it happened"

By Laughing Man (not verified) on 04 Feb 2011 #permalink

@ Marc A

The Casimir plates do create the effect. Because they conduct, that places a restriction on the electromagnetic modes that can appear between them (so yes, Steve, the separation matters). QM tells us that an unoccupied mode has a nonzero energy, and a nonexistent mode has no energy. The difference gives us the Casimir force. You need the plates to get rid of the EM modes.

Another really neat thing is that if you put an excited atom in such a cavity, you can affect its relaxation rate. If the emitted photon isn't supported, the atom won't decay.

Defying god (or not) just doesn't enter into the discussion. At all.

For those still kinda confused about the whole black hole/radiation/virtual particles deal, I'll suggest "The Black Hole War" by Leonard Susskind. He does a great job of explaining a lot of these ideas in terms a layman can understand. It does take a bit of patience to get your head wrapped around it. And even after reading it twice, I can't say I have my head fully wrapped around it, as I couldn't really explain it myself. But I'm understand a lot more of it now then I did a year ago before reading it. His other book "The Cosmic Landscape" is a great one too. His theory on "where our universe lies in existance".

Fantastic post btw! Saw this one through Pharyngula. I'll have to visit here more often, Ethan.

@15
"I've never, ever, ever ever heard that explanation for antimatter ever ever. Sorry, and not to call you a crank or liar, but I need a bit more confirmation on that idea. "

This is actually correct, I think. Again, check out Black Hole War by Susskind, it does go into a bit of detail on this. And you have to remember that "backwards in time" doesn't really mean you know, the particles traveling back to the 1800's. You have to take into consideration the whole four-dimensional spacetime. In "forward" time, particles spin a specific direction, and like particles always spin in like direction. When it's spinning in the opposite direction, its "going back in time". Imagine a video tape of a person walking. Playing it one way, you can easily see that the tape is going forward, as the person is making recognisable movements. Play that tape backwards, and you can again easily tell the tape is running backwards, because the person on the screen is walking backwards. But if you "watch a video tape" of particles, there's not really any way to tell if the recording is going forwards and backwards, unless you know before hand. Either way.. I don't recall all the specifics, but I'll see if I can look them up tonight and we can get some clarification on it.

Laughing Man is a troll you sillies :)

(hence, the name)

Thanks for the replies. I think our disagreement is simply my intuition (possibly false) telling me God doesn't play dice. The evidence seems to bear that reality, but I have a hard time believing that's the end of the story. You guys seem to have a better grasp of the actual science in question(I jumped to this board from Pharyngula), but I think you're underestimating my appreciation for the "realities" imposed by QM.

"Oh, and that "defy god" doesn't happen, since we have no evidence for gods.

the fact that you're able to distinguish the two things you say are equivalent makes them unequivalent in "some regard".
This is trivial but non-consequential here. Colors can be distinguished by wavelengths so they are nonequivalent (not the same). But equivalently they are equivalent (photons with wavelengths)."

Ok...I was using the example of God to get under your skin and make you think about what you're saying. I believe it still holds. Einstein didn't disprove Newton, and whoever comes up with a better explanation for why an anti-particle/particle "appears" at a certain(but not that certain, I know, Heisenberg) time and space wouldn't disprove QM. It would be a refinement. I think it's a legitimate question. It might never be answered, but we'll probably never detect other universes(wtv that means)...it doesn't mean there aren't any.

Yes...we perceive different wavelenghts of light as different colors, but those different wavelengths of light being emitted are different for particular reasons. We can explain why this is so...so that's not a very good example.
I guess I should make one thing clear before we go on. I don't believe in anything being actually random. I believe everything that seems random is simply pseudo-random(has an explanation).

I seem to be overstepping according to you. I'd love to know why. Should I just take a QM class and shut up?

Those who insist on treating the problem as unsolved because space and physical laws are involved here are missing a deeper point about the spontaneity of existence. We have no reason to believe a state of (absolute) nothing is privileged--it's not even clear if such a condition is possible. By contrast the existence of actual things does show that the existence of something is possible. People interested in a serious take on these questions should have a look at Adolf Grunbaum's exchange with Richard Swinburne over his article, "The Poverty of Theistic Cosmology" in the British Journal for Philosophy of Science: Grunbaum, BJPS

By Bryson Brown (not verified) on 04 Feb 2011 #permalink

Chuck Nance (No 26) is, of course, correct when he says, 'It is impossible for something to come from nothing.' And there is no observational or experimental evidence to contradict this. These 'particle-antiparticle pairs' have never ever been detected, and they certainly have never been shown to 'wink in-and-out of existence'. (Or perhaps you can tell us which experiment detected a single particle coming into existence from nowhere.) These particles are pure fiction - they are a wholly mathematical invention. I thought that physicists are supposed to rely on facts, not mathematics, to tell them what is possible.

If physicists cannot explain the Casimir effect without resorting to unsubstantiated creationism (which is exactly what asserting 'particle-antiparticle pairs wink in-and-out of existence all the time' is) then physics is in a very sorry state.
Tom (No 19) is most probably correct when he says, 'I think it's fair to say that the Casimir force is an electromagnetic force.'
I have a fundamental question: can the Casimir effect be (re)produced with plates of different materials ? As it seems that only metal plates are mentioned - what about plastic, glass, wood etc ? Because if there was some real vacuum force there it would certainly act on two wooden plates. And if you can't produce the effect with wooden plates - only metal ones - then it suggests that the force is indeed electro (and/or) magnetic in nature.

By Mark Robson (not verified) on 04 Feb 2011 #permalink

Mark Robson @ 73:

blah, blah, blah

Ah, another non-scientist here to explain science to us. Gee, too bad we went to all that trouble to get degrees in our fields, when all we had to do was ask Mark!

From Wikipedia:

In the original calculation done by Casimir, he considered the space between a pair of conducting metal plates at distance a apart. In this case, the standing waves are particularly easy to calculate, since the transverse component of the electric field and the normal component of the magnetic field must vanish on the surface of a conductor.

So, there was an actual reason to use metal plates. And if they are uncharged and non-magnetic, then there go the "electro (and/or) magnetic" rationales.

And as the description notes, the vacuum is not nothing; there are neutrinos in there, there are photons in there, etc. I'm not trained in the details enough to step through the math, but it is certain that all of these objections have been addressed in the literature.

Your problem, Mark, is that you are fixated on the idea that reality conforms to your personal feelings, as a sort of argumentum ad definition, to coin a phrase. It's like people saying Pluto has always been a planet, so if astronomers redefine the term 'planet' to exclude Pluto, then they don't understand the topic.

Chuck Nance (No 26) is, of course, correct when he says, 'It is impossible for something to come from nothing.'

that only sounds right because it's in English, not mathemathics. try proving it formally.

it seems to us to be right because we're using macroscopic-scale experience to intuitively grasp the world. but the world does not, fundamentally, cater to our intuitions; we know that quantum mechanics works in a number of ways that stand such intuitions thoroughly on their head before kicking them out the window --- without either opening it first or breaking the pane in the process. (tunneling FTW.)

why should we assume that a quantum-scale "vacuum" works and operates the way a macro-scale "vacuum" intuitively seems to? nothing else on those scales does. photons pass through two slits at once, electrons escape potential traps they simply don't have the energy to climb out of, nothing can be said with certainty to be specifically there and moving precisely thattaway at exactly this speed because those basic properties do not simultaneously exist, and things have "spin" even when they lack a spatial dimension in which to be "turning"... how is particle-antiparticle pair creation and annihilation any stranger, really?

we know quantum mech works. we've proven it, to the point that everyday technology now relies on it. the fact that it makes no sense to macroscopic intuition is one of which the universe seems supremely heedless. just give up trying to force it all to make sense on that level, because it won't.

By Nomen Nescio (not verified) on 04 Feb 2011 #permalink

Chuck Nance (No 26) is, of course, correct when he says, 'It is impossible for something to come from nothing.'

Technically, he's committing a logical fallacy. And so are you.

And there is no observational or experimental evidence to contradict this.

Well, we have never seen any "nothing at all", and there is no observational or experimental evidence to suggest that "nothing at all" can exist, let alone has ever existed.

These 'particle-antiparticle pairs' have never ever been detected

Ah. When you don't like the observation or the experiment, you assert that it didn't happen.

Because you, of course, are God, and magically know everything?

(Or perhaps you can tell us which experiment detected a single particle coming into existence from nowhere.)

What part of "particle-antiparticle pairs" -- as in, not single particles -- did you fail to understand?

These particles are pure fiction - they are a wholly mathematical invention.

Because mathematical inventions can have real effects?

I thought that physicists are supposed to rely on facts, not mathematics, to tell them what is possible.

Because mathematics is not factual? 2+3=5 is not fact?

Since they did indeed rely on facts about the real world, they are indeed describing what is possible about the real world.

By Owlmirror (not verified) on 04 Feb 2011 #permalink

I have a slight disagreement here, Ethanâ¦

I see what you did there.
.
.
.
"Now, that's what we know we can get, even from nothing. But there are many things we can't do, either practically or theoretically: violate charge or energy conservation, decrease the total entropy of the Universe, or figure out where our initially inflating Universe came from. (Yet!) But we definitely can get something for nothing; quantum field theory not only allows it, it demands it. But it remains to be seen whether we can get everything for nothing."
.
.
.
You can not get something FOR nothing.
You can however, get something FROM nothing.
There is a cost, there is always cost, and that cost is change.

While the differentiation is slight it does bare a significant relevance in the direction of a mindset.

Rush is right.
www.youtube.com/watch?v=096LhjGNNCk

"Nothing" can be described as 100% identical in totality as its only quality.
If all that exist is the same , all broken down to the minutest ingredient, then it is impossible to distinguish anything, a subset of nothing can not be distinguished from the whole of nothing.
There is nothing to compare, there is nothing.

From a Universal viewpoint âNothingâ was abundant.

Total âNothingâ can not be defined. There is no gauge to measure to acquire information or provide constraints from within "nothing".

The word nothing, CAN be defined: State of nonexistence; a condition of nonexistence, or the absence of any perceptible qualities.

All the gold in the Universe can't buy you "nothing".

And if I may add conjecture,

Space is created by mass, whether this be real or virtual.

âNothingâ induces virtual particles, Virtual particles create virtual space, Virtual space allows for the creation of real particles, Real particles allows for the creation of real space.
From a geometric viewpoint, Interaction is inevitable.

"God does not play dice" Einstein said, and if he had the anonymity of the Internet, he might have concluded (because he has no roll/role).

By Sphere Coupler (not verified) on 04 Feb 2011 #permalink

This may have already have been answered. The question arose that if a particle escaped a black hole through the hawking effect then doesn't the black hole gain the mass of the particle that did not escape? A: the black hole consists of both mass and energy . For the virtual particle-antiparticle to turn real and for one to escape the event horizon enough energy must come from within the black hole and approach the event horizon where the particle-antiparticle are forming to 1. Form the mass for both particle and antiparticle. 2. Give the particle or antiparticle escaping enough energy to escape just outside the event horizon. Since mass and energy can interchange due to e=mc^2 subject to various constraints (momentum, various particle physics rules,etc) the equilibrium between mass and energy will fluctuate to Allow for the smaller mass/energy of the black hole.

Space is replete with structure that supports all the physics we know about and the physics we don't know about. The question remains: Who created that structure and all the physical laws and structure in our physical universe?( And the big bang too.) I believe it is God. There is no logical contradiction in believing in God and believing in the laws of physics. We must just allow for God to intervene as needed for those things outside the realm of science such as the initial creation of space, mass/energy, and everything else we see in this physical universe.

Mike peralta;

If you need a God of the gaps to give you that warm fuzzy feeling to soothe your intellect, then that is fine for you, it just isn't using your intellect to the highest degree, it is a cop out, you choose not to investigate and if there is, was or will be a god, I think it would be very disappointed with your reasoning.

If we allowed for a god to intervene for those things we do not understand then we would never, ever make any progress.

By Sphere Coupler (not verified) on 05 Feb 2011 #permalink

Let me clarify. Belief in God and in the laws of physics or science are not logically mutually exclusive. I myself firmly believe in God and make substantial progress as a semiconductor modeling engineer. Creating many physical models using physics and math and the scientific method. I believe in science to describe the physical realm. I believe in God to explain the spirit realm and the creation of the physical realm -- since among other things science has no answers on how space, mass/energy came into existence in the first place. In fact science opposes creation of space/mass/energy due to conservation rules. My personal belief is seek truth no mater where it takes you -- if genuinely done it will lead to both God and scientific discovery --and that is true progress. Btw In case you are wondering i have a Ph D in physics from the univ of Arizona, BSEE , and a BS Applied Math. I love science. And I love God. But I love God more.

Well Mike, if it works for you then more power to ya.I was not wondering about your credentials, I can tell from your written speech that you have attained a certain level of accumulated education, though a Ph D, a BSEE and a BS is in no way an indicator of superior intellect.

You have your beliefs and if they are not harmful to others then I see no pressing reason for you to change,
Personally, I find it more acceptable to me and conducive to discovery to remain neutral in this matter of relying on a god for answers to the mysteries of life, so perhaps we could agree to disagree.

By Sphere Coupler (not verified) on 05 Feb 2011 #permalink

I am glad I don't believe in god, because then I don't have to look like I am talking out of my ass about shit I have no clue about to "disprove" theories that contradict my worldview. It just seems easier to just say, "You know what? I trust the science."

By Knative07 (not verified) on 05 Feb 2011 #permalink

The coupling of Diversity in thought is the key.

While some choose to take the established Academic route, their are others that excel didactically, just as there are those who can compartmentalise exploratory procedures, there are those who take neither a theistic nor an atheistic point of view.
The Maverick comes in many flavors.

Knative07;
Yes it's easy to say "I trust the science" and perhaps even easier to state one's belief in a god, however, the easiest route is a standard, it is the disturbance of the standard that leads to discovery.

I remain skeptical, yet work within protracted, fruitful models derived from the scientific method.

I trust that science is ever changing as time progresses.

By Sphere Coupler (not verified) on 05 Feb 2011 #permalink

I meant autodidactically,(my bad ;?) derived from,

(...link in buffer)

By Sphere Coupler (not verified) on 05 Feb 2011 #permalink

I am a Black Hole, but I do not eat - because I don't have to. I simply create my own lunch outside.

In my diary:

Looking back into my past I can see the creation of nothing from something.

If I look further back into the past there is more of nothing.

Therefore I must be the Creator of everything in the future.

http://arxiv.org/pdf/1102.0570:

Dust in the early Universe: Evidence for non-stellar dust production or observational errors?
Lars Mattsson

Observations have revealed unexpectedly large amounts of dust in high-redshift galaxies and its origin is still much debated. Valiante et al. (2009, MNRAS, 397, 1661) suggested the net stellar dust production of the quasar host galaxy SDSS J1148+5251 may be sufficient to explain the large dust mass detected in this galaxy, albeit under some very special assumptions (e.g., 'closed box' evolution and a rather high gas mass). Here it is shown that since accretion of essentially pristine material may lower the efficiency of dust formation significantly, and the observationally derived dust-to-gas ratios for these high-redshift galaxies are remarkably high, stellar dust production is likely insufficient. A model including metallicity-dependent, non-stellar dust formation ('secondary dust') is presented. The required contribution from this non-stellar dust component appears too large, however. If all observational constraints are to be met, the resultant dust-to-metals ratio is close to unity, which means that almost all interstellar metals exist in the form dust. This is a very unlikely situation and suggests the large dust-to-gas ratios at high-redshifts may be due to observational uncertainties and/or or incorrect calibration of conversion factors for gas and dust tracers.

Awakening article!
Imagine a gravity cannon made of an arrangement of blackholes creating a one-way tunnel of lagrangian points.
Particles and antiparticles pairs would be ejected (not captured) in a single direction, so we would have a sequential random output of particles or antiparticles that could be harnessed to the left or the right according to their nature and this way create a generator of matter and antimatter: unlimited matter and energy.
Also, during the casimir effect, why matter and antimatter pairs are not created and then desintegrate, liberating energy?
Also, yhere does the energy to transform energy of the void into matter comes?

could it be that occasional rays disturb the nothing like water surface divided by a advancing ship, and if the perturbation reaches a threshold, it can reach the energy levels required to forme matter?

Also, why the creation of antipairs would be short lived during the casimir effect? can't there be long lived antipairs that should accumulate?

@36: "Conservation of energy tells you this energy has to come from somewhere : the black hole."
First of all I find it misleading to state that the whole event is happening "into" the black hole; by definition, half the event is outside, otherwise no half of the pair could be emitted. If each half consume 0.5 MeV/c² to exist, the black hole provides only the energy for its captured half, and the outside provides for the other half. So precisely speaking, both the black hole and the surrounding space are loosing energy (but gaining matter), which means that everything is slightly more matterish-centripetous and less energetic-centrifuge. But that's just modes of the same "existence" thing, mass remains the same, only its distribution changes.

@39: are you saying that some sort of symmetry principle make a particle near the e.h. attracts its reflect, making it realer until it is really?

@44: "If an action can't be balanced within the limits set by Heisenberg, it just doesn't occur."
Why? What's the concrete mechanism? Which forces are acting and reacting to forbide this event?
Are you saying because all the parallel universes are all occupied with antimatter, there is no easy room for more existence of antimatter, as easy and thus probable than existence of the opposite: entry of matter.
Thus, when an unavoidable future of some particle has in every possibility the same outcome, it becomes real and calls a simultaneous exactly similar quantum of reality that becomes anti-real. There is a mysterious symmetry. Why stable things would emerge from chaos? Become stability and chaos are the same? Things happening because of this or its exact contrary, it's the same result. Every thing has two possible roots. Branching pasts, branching futures...
Symmetry of yes and no, good and bad, nothing and something, matter and antimatter.

@45: you could as well be describing the concept of avatars of a single bigger entity, like a toolkit "particles + energy" with several isotropes. Why do the fundamental states of matter (particles) have several appearances?

@46: are you saying that it is possible to distinguish things through the mind that aren't really there? Like a strange illusion, impossible but neverendingly convincing?

@47: not Newton but Anaxagore.

@56: are you saying that common sense and philosophical logic aren't dealing about the same reality than science? While I agree that they not equivalent, some subsets can have common grounds with science.

@75: «things have "spin" even when they lack a spatial dimension in which to be "turning"»
how do you know they lack spatial dimensions? what about tiny looping dimensions created by their own mass?

«just give up trying to force it all to make sense on that level, because it won't.»
I don't understand the rationale leading to this conclusion.

Confirming that Mr. Physicist dude doesn't believe in any conservation laws.

That, or he's just playing specious semantics games.

By sherifffruitfly (not verified) on 05 Feb 2011 #permalink

From Dr. RC Sproul:

"Scientists are fallible and may occasionally make arrogant statements that go far beyond the realm of their own expertise.

Recently I read an essay by a well-known Nobel Prize winning physicist (whose name will remain unstated so as not to embarrass him) who argued that the idea of âspontaneous generationâ be abandoned in science once and for all. Spontaneous generation means that something comes into being with no cause. It comes from nothing. So far, so good. I was pleased to see a scientist debunk the myth of all myths, that something can come from nothing. This myth is still pervasive in the scientific community with respect to âchance.â Chance is given credit for creating the universe. However, such a prodigious feat is beyond the capabilities of chance. Why? Chance can do nothing because it is nothing. Chance is merely a word we use to explain mathematical possibilities. It is no thing. It has no power. It cannot produce, manage, or cause anything because it is nothing. It is spontaneous generation by another name.

I was glad the physicist repudiated spontaneous generation. My gladness abruptly turned to astonishment when the scientist said, âWe must have a new model. We must speak in terms of gradual spontaneous generation.â I couldnât believe what I was reading. âGradual spontaneous generationâ? How can something gradual be spontaneous? How can something spontaneous be gradual?

Our scientist wanted to debunk the myth that something can come suddenly from nothing and replace it with a better myth that something can come gradually from nothing.

I use this illustration only to show that even the most astute scientists can nod. They can fall asleep at the switch and be suddenly very unscientific in their pronouncements. To believe in gradual spontaneous generation of anything is to leap not by faith but below faith to credulity. Such a concept defies both aspects of the scientific method: rational deduction and empirical observation. Not only is the idea in violation of reason (breaking the Law of Contradiction), but it is impossible to observe empirically. What microscope or telescope is strong enough to observe anything doing something gradually spontaneously?"

Don't know if this is exactly what you are discussing, but it's good truth and logic.

I was glad the physicist repudiated spontaneous generation. My gladness abruptly turned to astonishment when the scientist said, âWe must have a new model. We must speak in terms of gradual spontaneous generation.â I couldnât believe what I was reading. âGradual spontaneous generationâ? How can something gradual be spontaneous? How can something spontaneous be gradual?

Mike, I thought you were a pantheist/deist, but the website that your name links to is a weird collection of mp3s about Jesus, Hell, and the Rapture. I think you're kinda misrepresenting yourself, because the claims that Jesus was more than human, Hell exists, and that a Rapture will occur are all unsupported by what we currently know about reality.

Please don't respond to this with a dump of verses from your preferred translation of the Christian Bible, tia

I'm not expert in physics, but I have studied logic, and I hate, hate, hate, hate it when people use 'logic' to insist that the cosmos must be one way or the other. I hate it.

Logic, real logic, alone cannot tell us whether something can come from nothing or not, just like it couldn't tell us if the orbits of the planets are perfect spheres or if the sun orbits the earth. That's not what logic is. When people run around acting like it can, like we can sit in a room in our house and reason out the laws of nature without actually looking at reality, we end up with incorrect conclusions that cloud our understanding for centuries or more.

@53 Laughing Man

When you get life from non-life, then we'll talk, Ethan. M'kay?

Life is made out of atoms, which are not alive in any way. There you go.

By Citizen of the… (not verified) on 05 Feb 2011 #permalink

adsense hack @ 91 - you've got a definition mismatch.

Not 'spontaneous' in the sense of 'sudden', but rather in the sense of 'happening without an external cause'.

I will ask more clearly this time: can someone direct me to a reference - book, article or website even - in which a scientifically valid, verifiable and repeatable experiment demonstrated ''particle-antiparticle pairs wink[ing] in-and-out of existence . . .''
My 'personal feelings' on this are that, as I was told decades ago, matter (and energy) cannot be created or destroyed - so I didn't think that it was possible for ''particle-antiparticle pairs [to] wink in-and-out of existence all the time.'' So I'm glad that Dr Siegel, NJ (No 74), Owlmirror (No76) and others have seen verifiable proof of this - or else why would they believe such a thing - and so I would like to see this proof for myself. I also have related questions:
1) is there a minimum density of vacuum you need for the particle-antiparticle pairs to appear ?
2) given that in any time interval there will be more shorter time periods, both sequentially and concurrently, is it more likely that you get more heavier (greater mass) particle-antiparticle pairs ?
3) are there any plots of the mass/charge/speed distributions of the particle-antiparticle pairs ?
4) are there any differences of mass/charge/speed of the particle-antiparticle pairs in Hawking radiation compared with the lab measurements ?
5) is there an increased rate of particle-antiparticle pair production the less dense a vacuum becomes ?
6) surely some particle-antiparticle pairs must appear in the vacuum of particle accelerators - does this cause many problems, and if so: how are they overcome ?

@NJ (No 74): Yes, I did read that the metal plates were uncharged and non-magnetic (so that there were no electro (and/or) magnetic forces acting) but if Casimir had used an insulating material for the plates he would also calculate the magnetic field on the surface to be zero. There is no magnetic field on a glass plate - is there ? I was simply asking (again): can the Casimir effect be (re)produced with plates of different materials ? My thinking was that if the Casimir force only acts on metal plates then a further explanation is needed to explain how the particle-antiparticle pairs only act on metals and not on anything else. Wouldn't that be an entirely new type of force - a force that only acts on uncharged and non-magnetic metals ?

@Owlmirror (No76): Yes you're right - I should have said 'particle-antiparticle pairs' and not 'single particle'.

However you clearly don't understand maths. Your equation: 2 + 3 = 5 is, of course, fact - mathematically and in the real world, but the equation 2 - 3 = -1 is only a fact mathematically. You simply can't do that in the real world - try it: start with any 2 identical objects, then take 3 of those objects away to leave you with minus 1 of the objects. Therefore the perfectly accurate mathematical equation cannot be applied to the real world - the maths has failed and you can't use that mathematical rule to tell you what the real world is like.
Next try 2a x 3a = 6a^2. Let 'a' stand for apples: take 2 apples and multiply them by 3 apples - not only will you not get 6 apples squared, you will not even get 6 apples, you will only ever get the 5 apples you started with. The maths has failed again and you can't use the mathematical multiplication rule to tell you what the real world is like.
What about imaginary numbers ? e(exp)[ i x theta ]: take the number 'e', raise it to the power of 'i' (the imaginary number) x (the angle) theta. What do you get, and what does that represent in the real world ?
Maths is a very valuable tool that can be used to confirm that a theory is correct, but mathematical rules alone are never proof that something can, does or will happen in the real world.

By Mark Robson (not verified) on 07 Feb 2011 #permalink

#94

Life is made out of atoms, which are not alive in any way. There you go.

Fine. Then you're not alive, either. Give me all your stuff since you don't need it not being alive and all.

By Laughing Man (not verified) on 07 Feb 2011 #permalink

What if I posit that the Cashmir effect and Heisenberg's Uncertainty Principle apply to physical things. Therefore, something coming from nothing means that it doesn't have a physical cause. However, that doesn't mean that it can't have a non-physical cause (such as mind, or something else).

By Brian Dean (not verified) on 08 Feb 2011 #permalink

Ethan says, "This isn't the gravitational force or an electromagnetic force, but a force due to empty space itself." Hmm, these words are not precise enough.

Of course his meaning depends on your definition of "empty space"; but quibbling about the meaning of "nothing" is not my point.

Wiki says, "When this field is.. studied using quantum electrodynamics, it is seen that the plates do affect the virtual photons which constitute the field, and generate a net force... The Casimir effect can be understood by the idea that the presence of conducting metals and dielectrics alters the vacuum expectation value of the energy of the second quantized electromagnetic field."

So is the Casimir field a QED field, the second quantized electromagnetic field?

Thus, does Ethan more precisely mean that the Casimir field "isn't the (classical)gravitational force (of Einstein) or an (classical) electromagnetic force (of Maxwell), but a (quantum mechanical) force due to empty space itself (which can be expressed by QED (alone), or perhaps also with QCG and some future Quantum Gravity)"?

Ethan or someone, please further clarify, the meaning is unclear. Thanks.

Chuck Nance, Marc A, Nomen Nescio, Mark Robson, please please please listen to Lyra@93:

I'm not expert in physics, but I have studied logic, and I hate, hate, hate, hate it when people use 'logic' to insist that the cosmos must be one way or the other. I hate it.

Logic, real logic, alone cannot tell us whether something can come from nothing or not, just like it couldn't tell us if the orbits of the planets are perfect spheres or if the sun orbits the earth. That's not what logic is. When people run around acting like it can, like we can sit in a room in our house and reason out the laws of nature without actually looking at reality, we end up with incorrect conclusions that cloud our understanding for centuries or more.

You can't tell you anything about the world that you didn't put in. A priori reasoning is by definition tautological. You can get something for nothing in a vacuum, but not in logic.

You simply can't reason the universe into "behaving properly." The universe will do what it does and all you can try to do is keep up. And that's why this sentiment from Marc A is very unscientific:

I don't believe in anything being actually random. I believe everything that seems random is simply pseudo-random(has an explanation).

WHY don't you believe in randomness? It seems to me that this is an ontological presupposition -- something you believe to be true of the universe purely because it makes more sense in light of the framework you use to make sense of the universe. But why are you so sure of your framework? Why make assumptions when it's so much more fun to be surprised? (OK, so it takes a little more work to consider multiple possibilities than to decide that one MUST be true...the solution is to just not be lazy.)

laughingman@53:

When you get life from non-life, then we'll talk, Ethan. M'kay?

I think we can agree dried fruit is non-living. Eat some dried fruit. Your body will use the material from the dried fruit for cellular growth and division. Living human cells from non-living organic matter. Easy as pie and almost as delicious.

Mark Robson @ 97:

I was simply asking (again): can the Casimir effect be (re)produced with plates of different materials ?

In your preferred search engine, type "Google Scholar".

In the search box, type "Casimir effect dielectrics".

Examine references.

The topic is waaaaaaaaaaay out of my field, but the references shown appear to support my contention back in #74:

it is certain that all of these objections have been addressed in the literature

It seems clear from your reply to Owlmirror's post @ 76 that you are stuck at the level of concrete thinking, which explains a lot. Any adult who would use "2 apples times 3 apples doesn't give you 6 apples" as an argument is pretty much beyond hope of serious thought.

NJ@102:

It seems clear from your reply to Owlmirror's post @ 76 that you are stuck at the level of concrete thinking, which explains a lot. Any adult who would use "2 apples times 3 apples doesn't give you 6 apples" as an argument is pretty much beyond hope of serious thought.

I was about to make an apples^2 joke, then I went and looked at Mark's post and realized he had already talked about apples^2 and wasn't joking.

That's pretty scary.

Mark, if I have 3 groups of 2 apples, how many apples do I have? How did you get the answer? Multiplication makes perfect physical sense if you don't try to force it not to (as you did in your example).

On the subject of imaginary complex numbers, the complex plane is basically the same as R^2. Every complex number corresponds to a point on the coordinate plane.

I can't tell you what e^i*theta is if you don't tell me what theta is. e^i=-1 makes perfect physical sense: it's (-1,0) on the coordinate plane. The derivation of interpretation of the term "e^(i*theta)" is actually quite beautiful.

But besides simply being points on the coordinate plane, complex numbers can be used to quite accurately model the flow of heat through a material or the transmission of vibrations through a medium. Try doing that by adding integers.

OKThen@100:

Thus, does Ethan more precisely mean that the Casimir field "isn't the (classical)gravitational force (of Einstein) or an (classical) electromagnetic force (of Maxwell), but a (quantum mechanical) force due to empty space itself (which can be expressed by QED (alone), or perhaps also with QCG and some future Quantum Gravity)"?

Ethan or someone, please further clarify, the meaning is unclear. Thanks.

Imagine a tub full of plastic balls of different sizes. In it is a box with holes on all sides -- the holes are big enough for the smaller plastic balls to get in, but not the larger ones. We'd expect the pressure on the outside of the box to be greater than the pressure inside the box because the box can only let a particular fraction of the nearby balls inside.

The balls are virtual particles and the box is the Casimir plates. The net inward pressure on the box is the Casimir effect. The big difference is that with the real Casimir effect it's not the "size" of the virtual particle (it doesn't have a size in any straight-forward sense) but the wavelength -- only certain wavelengths can "fit in the box." Obviously, this is a really rough analogy, but does it help at all?

Follow up:

The trick works fine if you just consider virtual photons. Imagine a universe that looks like this:

::::::::|...|::::::::

Where dots are virtual photons (they'll act just like regular photons) and the pipes are the Casimir plates. The virtual photons have essentially random velocities relative to the plates. There's fewer photons between the plates because only certain frequencies of photons can exist between the plates (similar to why a guitar string makes a particular note).

The virtual photons that hit the plates are absorbed by the electrons in the plates, and when that happens the plate also picks up the photon's momentum -- basically, it gets bumped. The photons get re-emitted in a random direction. This means the plate gets bumped again, but this time in a random direction. Since the direction is random for every bump from an emission, the emission bumps cancel each other out. But the bumps from the absorptions aren't random -- there are two bumps from outside the plates for every one bump inside the plates. This effect provides a net force pushing the plates together.

Looking at it this way, the force pushing the plates together is basically electrostatic repulsion, the same reason you can't walk through walls.

I'm a bit of a dilettante, real physicists please correct anything I flubbed.

Thanks Dan L. and yes yes.
But my question remains, which type of phenomenon is Casimir effect:
-- QED (involving photons real or virtual)
-- QCD (gluons real or virtual)
-- Quantum Gravity (gravitons real or virtual)
-- Quantum Weak (W, Z real or virtual)
-- some new (force) quantum phenomenon

Silence of Ethan and the experts suggest that Casimir effect is a QED phenomenon.

OKThen;
I wrote a couple of pages of explanation, then realised that someone reading what I wrote would be more confused than if I just said;

In the Casimir Effect it is realized that it is a coupling effect of QED to QFT by the difference in quantum amplitude/probability amplitude(density) of zero point energy.

Yet, even that is general and far from complete.

By Sphere Coupler (not verified) on 09 Feb 2011 #permalink

Sphere Coupler
107 and 108 do help.
Though I don't quite understand how QFT is different than QED but I'm researching QFT online, starting with wiki.
Thanks.

I'm a laymen. Ahhh! My field of choice is computer programming. We use pseudo-random numbers. Real world turbulence can produce 'true' randomness. I've always not trusted this idea that it's 'true'. Maybe it's because when I first started with computers I thought that the pseudo-random numbers were truly random. Hey, I was a new student and can you blame me that I thought rand() gave random numbers? Anyway, if everything in the universe is connected, how can something be random unless it's isolated from everything else? If you had access to the entire universe and could monitor all of it then nothing would be random unless there were elements of it completely isolated from everything else in the universe. How is that possible? Anyway, the complexity of turbulence prevents us from predicting it, but if that's our only reason for claiming it's truly random, then that's a lacking explanation. If we were god or had perfect knowledge then we could understand the exact positions of all particles and forces and nothing in turbulence would be random to us since we could perfectly predict the position of every existing thing.

And one more thing. In my physics 101 class we were taught nothing can be created or destroyed. If this is true, then how can particles appear and disappear at quantum scales? How can something appear from nothing or become nothing without being created or destroyed? How am I confusing this?

Lastly (my final comment before I leave), why do the even replies have a selection background and the odd replies do not? This confused me when I was reading the replies because I kept thinking that the odd replies were responses to the even replies. I don't know why it's this way, but I just thought I'd leave this (somewhat critical) comment about it.

I'll take a crack at a couple of these, Jon...

why do the even replies have a selection background and the odd replies do not?

I've always assumed it was for readability. The comment threads are linear, instead of the branching types found elsewhere, which means they can get long. It also explains the common use of posters handles and comment numbers in replies.

In my physics 101 class we were taught nothing can be created or destroyed.

You kinda answered this in the prior comment:

when I first started with computers I thought that the pseudo-random numbers were truly random.

Things tend to get simplified in lower-level classes. One of the things that makes QM so hard to learn is that it is often counter to our macro-scale intuitive sense of how the world works. But the results speak for themselves.

Sphere Coupler # 110

The Wiki manifestation link is very helpful. Not just the manifestation section; but the whole entry.

Halfway through my read, I came to the conclusion that all particles are virtual, then in the Feynman diagram section it says, "It is sometimes said that all photons are virtual photons."

As well I suppose, it can be said that only virtual gravitons have been observed and maybe real ones can never be observed.

Thanks again for all the links.

In my physics 101 class we were taught nothing can be created or destroyed. If this is true, then how can particles appear and disappear at quantum scales? How can something appear from nothing or become nothing without being created or destroyed?

Simple: they do not appear from "nothing"; they appear from the zero-point energy, the quantum vacuum. This is not a true vacuum because it is "stuffed" with energy, and this energy occasionally realizes as virtual particle-antiparticle pairs. That is, they are not "appearing" or "disappearing," but simply changing form. Heisenberg, who actually did know something of philosophy, thought this might be the hule prote of Aristotle.

Commeht 117 said:
"Simple: they do not appear from "nothing"; they appear from the zero-point energy, the quantum vacuum. This is not a true vacuum because it is "stuffed" with energy, and this energy occasionally realizes as virtual particle-antiparticle pairs. That is, they are not "appearing" or "disappearing," but simply changing form. Heisenberg, who actually did know something of philosophy, thought this might be the hule prote of Aristotle."

Then we're not getting something for nothing. This would undermine the whole premise of this article - the idea that things are created from nothing. If things are in fact being created from something then there's no instance of getting something from nothing and therefore I don't see the point to this blog post.

And another point. At the end of the blog it's stated that having more matter than anti-matter means they haven't combined to result in ZERO and therefore we have something rather than nothing. But the problem with this is that in order to have 'created' the matter and anti-matter particles you had to have something, right? So before all of these matter and anti-matter particles were created we had SOMETHING. So ZERO is SOMETHING. It's only when they recombine that you get ZERO, but ZERO is not NOTHING! If NOTHING is ZERO and ZERO is something, then what we have no is somethig then what we have now is also NOTHING!

Sorry I mistyped. If what we have now is SOMETHING and ZERO is SOMETHING (because you can't create the matter and anti-matter particles without something) and NOTHING is ZERO then what we have now is also NOTHING!

OKThen

If I may point out a slight but critical correction...

You said;
"As well I suppose, it can be said that only virtual gravitons have been observed and maybe real ones can never be observed."

Truth of the matter is, virtual gravitons have been calculated...not observed.

conjecture;
Slightly off topic, It may be realised that SUSY is not a companion sparticle as the community has come to distinguish the theory, though it may very well be a precursor value of the virtual field from which a virtual particle arises, inducing real space, from which real space enables real particles.

But at this point that evaluation may be meta-physics, though I somehow *feel* and think SUSY is valid in some respect.

If the above has any merit then it will take a great deal more initial collision of the center of mass-energy to produce such a field.

By Sphere Coupler (not verified) on 28 Feb 2011 #permalink

Then we're not getting something for nothing. This would undermine the whole premise of this article - the idea that things are created from nothing. If things are in fact being created from something then there's no instance of getting something from nothing and therefore I don't see the point to this blog post.

I agree with comments 34 and 122. A pre-existing vacuum consists of space and time, both of which can be measured and, per Einstein, warped, slowed down, shrunken, expanded, etc.

This is not nothing, in the sense that people mean when they state "out of nothing, nothing comes."

So it seems like this blog post is essentially saying: "You CAN get something from nothing. Just start with a little something, see, and then..."

You may want to re-word that as getting something from something. In all the theory's like martin says you just need a little something. So something from nothing is impossible, in science in math, which neither can exist apart from each other, if so you negate the basis of both. The "components" themselves are something, i think the venture involves the aspect of how our system came about and what the universe is actually doing and what it may do is the basis of the theory's no one has ever suggested that everything started as nothing, that would be ludicrous.

To me there is a flaw in the logic resting on the Heisenberg's Uncertainty Principle stated above as paraphrased here: that you can't know a quantum state's energy exactly for a finite duration of time, therefore at short time intervals, there are large uncertainties in the energy of a system. So, the energies are large enough that particle-antiparticle pairs wink in-and-out of existence all the time.

If we are trying to answer the question "can we get something from nothing," isn't energy, a presupposed entity, something? Is energy not just a book keeping system about how things tend to relate to one another? What is time? It is the interval between events. What "events" if there is nothing? Therefore it is nonsensical to apply HUP to this situation. You are presuming that the area of "nothing" is surrounded by something. This does not answer our question about the origin of matter or energy.

No. But that "vacuum", with laws, 3 + 1 dimensions and possible curvature, laws etc, isn't really "nothing" and shouldn't be called that in the first place. It is "something", real nothing is the absence of all being, no laws or propensities, or even geometric properties like number of dimensions etc.

Indeed, given the implicit formability and existence - even if we call it "virtual" - of the particles we find, it's really the same existence question anyway as for the tangible universe. So that so-called-vacuum and its existence, especially as existential selectivity (the old "why the universe like this instead of some other way" - not just "why is there something instead of nothing" overall), stands in need of justification, of explanation.

Note also, that this "vacuum" needs "time" to define things like "chance that creation of a universe will occur per time unit per cubic volume of this space" etc. Hawking's celebrated analogy of a sphere with a north pole and "no further north" is misleading, since causal connection and not being able to graph continued past history is the real question. And again, the real issue is the existence of that "entity" even taken as a whole, not just whether it needs to have all causal chains resolved within itself. (Science writers, just FYI: a lot of what you write is philosophically challenged. And BTW this means "philosophy the conceptual framing process" not the popular red herring of a speculative subject matter per se. Not an insult, OK, just tighten up.)

And how did you do this? You had to set up an experiment using your brain, provided you have one, and try to show that it all came about with no help from you (intelligent life). Plus there is something. There's air. Something coming from nothing? Good job. Good job

As per the linked article: "Empty space is not really empty".

From that, this dishonest blogger infers something coming from nothing. Hahahaaaa

I find it rather comical of how blind and desperate the faith of the militant atheist is. This particular case is one where he distorts scientific theory to further a political - albeit futile - cause.

You're an embarrassment. Stop tarnishing science with your stupidity.

I would like to see you debate Craig. Put your money where you mouth is.

By John Smith (not verified) on 04 Sep 2011 #permalink

JS@129:

I find it rather comical of how blind and desperate the faith of the militant atheist is. This particular case is one where he distorts scientific theory to further a political - albeit futile - cause.

And a quick scan of the original post finds no reference to either atheism or politics.

So how on Earth did Johnny get these ideas? From the voices in his head, of course! I am led to believe that there are efficacious meds for this.

You're I'm an embarrassment. Stop tarnishing science the internet with your my stupidity.

FIFY.

I can follow the discussion for the most part, have some opinions, can in no way offer logic to argument...but thoroughly enjoyed it! Thank you. :)

"(because you can't create the matter and anti-matter particles without something)"

It's not creating matter and anti-matter particles. It's creating virtual particles that, as far as the universe is concerned, doesn't exist.

It's easy to create something that doesn't exist from nothing.

"For example, take a box and empty it, so that all you've got is some totally empty space, like above. An ideal, perfect, empty vacuum. Now, what's in that box?

Did you guess nothing? Well, it turns out that empty space isn't so empty."

Doh! That is not nothing. There was box and little minute "things" that u cannot see. There is always something there ...

... as are ur later examples.

"It's not creating matter and anti-matter particles. It's creating virtual particles that, as far as the universe is concerned, doesn't exist.

It's easy to create something that doesn't exist from nothing."

This comment from "Wow" - is plain silly. Something already exists to create what you termed as "virtual" particles. It is not from nothing.

And it is impossible to create something from nothing.

Ok, I will admit that most of the language spoken here is waaaaaaay above me and I have no experience or education with any of this. So, allow me to make my comment from a very "average Joe" perspective. I am intrigued by the idea of actually being able to get something from nothing, even that nothing is in itself "something". My issue with the cardboard box idea or two metal plates is that you still used something to get something. I cannot and may never believe that if there was at one time complete zero, nothing, not a vacuum or particle or force or microscopic thing or ANYTHING...then nothing would ever come out of that. I would love to hear someone sometime explain how complete and utter nothingness can create anything without something external causing a vacuum or implosion or matter/antimatter etc. I may be an idiot but I can with all my heart say that if you put nothing into a time capsule and bury it for a gazillion years then unbury it, there will be nothing. If you find something then SOMETHING had to be inside that time capsule whether seen or not. The trick might be the time capsule itself causing something to happen but you get my point. No matter how much time passes you simply cannot get something from nothing and if you do, something made it happen. Again, Im an average joe so I cannot debate this intelligently, and I hate to beat a dead horse, but no matter how you slice it, absolute nothingness produces...absolutely nothing.

By Robert Harvey (not verified) on 04 Nov 2011 #permalink

So in other words, vaccuum fluctuation models made no advance over the arguments put forward by those who believe in God. The "something out of nothing" is really just fancy language but not literally something out of nothing. The origin of the universe still seems to be best explained through a metaphysical route. I'm just saying, let's be honest with ourselves. If it's not a fact indicating something from nothing, then don't call it that; no doubt there will be those who will be mislead into believing the wrong thing.

> So in other words, vaccuum fluctuation models made no advance over the arguments put forward by those who believe in God

If by "other words" you mean "wrong words", then yes.

Casmiir effect. It's been measured.

That's more than any God botherer has managed with god, miracles or JC.

"This comment from "Wow" - is plain silly."

Nope, that comment from me was accurate.

You just think it's silly.

"Something already exists to create what you termed as "virtual" particles."

Space with nothing in it existed.

But space with nothing in it is the nothing you're creating virtual particles IN, not WITH.

> I cannot and may never believe that if there was at one time complete zero, nothing, not a vacuum or particle or force or microscopic thing or ANYTHING...then nothing would ever come out of that.

That's fine. You're not required to. The Universe will continue to exist.

"I would love to hear someone sometime explain how complete and utter nothingness can create anything without something external causing a vacuum or implosion "

I would love to hear someone sometime explain how something external existing can create something from complete and utter nothingness.

To hear the explanation, you'll need some serious postgrad work on theoretical physics.

It's possible to do so.

Feel free.

But lets hear you explain your alternative theory: how does something exist when there's complete and utter nothingness? How could it create something when there's complete and utter nothingness?

"Doh! That is not nothing. There was box and little minute "things" that u cannot see."

No, there's a lot of air that you can't see and nobody calls it "nothing".

That nothing in there contains nothing that it is possible EVER to see, since it doesn't exist for long enough to change the universe one iota.

Wow, I think peoples points are that it's very possible that our current understand of "nothing" is just wrong. Or at most, misleading. Saying that we can, in absolute certainty, observe a perfect void of all reality is like a person back in 1532 that humans will never fly.

The problem with saying the math proves that Physics allows something to come from nothing is that math & physics are something. Especially when we know that our current Physics model breaks down in things like Black Holes & the Big Bang. We understand that there must be larger laws we are unaware of to explain those events. The problem is that that simply takes us another step back. How did those laws form? I think that trying to say we can demonstrate that reality came from (true) nothingness, ie: no physical reality, no laws, no physics, no math, is just as ludicrous as saying God just made it. The only difference between the two is that people who believe in metaphysical reality are willing to admit that there is no FINAL physical explanation, It doesn't make then disagree with science. It may , however, make them disagree with people who say "I can explain it all".

We don't know the true nature of "nothing". We don't even know if it really "exists". Likewise, we don't the true nature of "god", or even if he really "exists". Therefore, we can't absolutely prove, or absolutely disprove, anything, we have no place to start. The above so called proof requires the prior existence of quantum effects, and forgets that there is no absolute proof that the quantum effects existed before the big bang. Yes, there is also no absolute proof they did not exist, but that's the problem, there is no absolute proof. Anything less than absolute proof may be highly suggestive of an answer, but it will never be able to settle the question once and for all due to the holes left in it by not being absolute.

Both points require belief and faith because they don't yet have enough facts to become absolute truth, and are therefore much more philosophical/theological positions, than they are scientific ones. This endless foolishness of using science to try and prove or disprove theology or atheology is doomed to failure because science cannot and will not disprove god's existence until we know everything about everything (because as long as there is one thing we don't know he will have a place to hide).

And of course, once we do know everything, we will BE god...

By Amused observer (not verified) on 18 Nov 2011 #permalink

We DO know the true nature of Nothing.
The simple equation Nothing + nothing = nothing is understood by any 1st grader.

Space is not nothing and that is clear to any physicist. Principles are also not nothing as one principle merged with another principle can make a new principle.

This isnt semantics..its bafoonery and completely transparent when used by atheists because the driving force behind all their articles are to eliminate God.

Somehow physicists have proclaimed themselves priests of the universe---yeah, like Im gonna learn the meaning of life from a bunch of nerds who spent high school lunch period with a grilled cheese mushed into the head and apple sauce in their hair.

These guys have officially left the building with the multiverse theory. At least they finally admit the odds of our universe. But now their odds include universes where 10 million suns are all lined up to spell "This is the awesome multiverse I created to avoid worshiping God"--because like it or not-- thats what an infinite amount of universes will produce...including one with a God.

If people thought logically and without bias, no one would ever listen to these guys again. They are the freakin OJ jury on steroids. Whats next elephants on unicycles? The Mind of God has always the simplest explanation for reality..you have to be a drug addict, murderer, fool, hyper arrogant, severe daddy issues, or a physicist to deny what is plain to everyone else in history.

Someone once said they know that something created the universe and that something was nothing. Now, it seems they want to redefine what Nothing is so its not actually nothing

ok im trying to understand this stuff but i have a question: can u posibly email me a list of all posible things u can make from nothing and maybe how to make each of these things if ANYONE can do this it would be great i hope u all get jobs as the best phisisists in the world and find out if its posible to make everything from nothing

By Elijah Adams (not verified) on 19 Dec 2011 #permalink

It doesn't last very long, Elijah.

Yeah, how can ANYTHING come from nothing?

If there are actually minute particles and anti-particles popping in and out of existance at a nearly unmeasurable time scale, does the box really ever have nothing in it? In reality, it sounds like "nothing" has been redefined to mean "lots of immensely small particiles that hop in and out of existance in our time and space."

By SimpleSkeptic (not verified) on 13 Jan 2012 #permalink

"Did you guess nothing? Well, it turns out that empty space isn’t so empty" .... well then that isn't nothing is it.....?

Yes, it is nothing. Any measuring device possible even theoretically to build would read nothing there.

It's the only nothing it's possible to get in a universe with space and time.

we cannot create nothing because we live in something (space and time)

That's right 'Simona' and it has physical properties, otherwise gravity would not work, there is something.

However, when the audience are not physicists, they will be using this definition of nothing:

noth·ing/ˈnəTHiNG/
Pronoun:
Not anything; no single thing: "I said nothing".
Adjective:
Having no prospect of progress; of no value: "he had a series of nothing jobs".
Adverb:
Not at all: "she cares nothing for others".
Synonyms:
pronoun. none - anything
adverb. not at all - none - never - not a bit - not in the least

Which EVERYONE has come across.

If you're talking to cosmologists and theoretical physicists, you're talking about mathematically nothing.

Which nobody has EVER seen.

The topic of the thread is pretty specific about what the nothing it talks about.

And it shows that the "common sense" idea that "you can't get something from nothing" is wrong. Ergo, it is no proof in and of itself that God exists.

And, since we don't have any absolutely nothing around, we can't prove observationally the Big Bang, but we can show that it is possible from the physics we know about to get something (the universe) from that nothing (absolute).

Not even that level of proof is available to those who wish to put a magical Creator in the limelight.

"And it shows that the “common sense” idea that “you can’t get something from nothing” is wrong."

No, Ramona talked about something else, that you can't create nothing to begin with, so it is hopeless to start saying that something came out of 'nothing'. The moment you can shield of the inside from that vacuum box off from gravity that your Vacuum would be more empty, but still it there would be something, Aether. That is what Einstein also said:

“Thus, once again ,,empty” space appears as endowed with physical properties, i.e., no longer as physically empty, as seemed to be the case according to Special Relativity. One can thus say that the Aether is resurrected in the General theory of Relativity, though in a more sublimated form.”

A. Einstein, Grundgedanken und Methoden der Relativitatstheorie in ihrer Entwicklung dargestellt, (Morgan Manuscript) Einstein Archives 2070.

and

“According to the general theory of relativity, space without ether is unthinkable; for in such space, not only would there be no propagation of light, but also no possibility of existence for standards of space and time (measuring rods and clocks), nor therefore any space-time intervals in the physical sense.”

A. Einstein, Aether und Relativitaetstheorie, Berlin: Verlag von J. Springer, 1920.

She said 'we' as in 'we humans' who live 'in' SpaceTime. We can not make the physical stuff that applies gravity to go away, we can't create an absolute void. Yes we can clear all the visible matter out of a box and create a 'vacuum' but at the end of the day that box is still filled with an Aether that could fluctuate.

"She said ‘we’ as in ‘we humans’ who live ‘in’ "

Who the hell says we created the universe?

She's as nuts as you are...

"Who the hell says we created the universe?

She said:

"we cannot create nothing because we live in something (space and time)"

Wow, are you now also hearing voices?

Perhaps it is the time to look for help to treat your illness.

Indeed, like I said "who the hell says we created the universe".

Nobody.

How do you manage to prove me right then pretend you've found a gotcha?

Than why do you feel the need to accuse her of being nuts?

At least you're admitting you're nuts.

Perhaps I'm nuts for debating with you, but not when I talk about Aether:

“According to the general theory of relativity, space without ether is unthinkable; for in such space, not only would there be no propagation of light, but also no possibility of existence for standards of space and time (measuring rods and clocks), nor therefore any space-time intervals in the physical sense.”
A. Einstein, Aether und Relativitaetstheorie, Berlin: Verlag von J. Springer, 1920.

The aether has been disproven by the Michelson-Morely experiment.

Even Einstein thinks you're wrong:

"All our attempts to make ether real failed. It revealed neither its mechanical construction nor absolute motion. Nothing remained of all the properties of the ether except that for which it was invented, i.e., its ability to transmit electromagnetic waves. Our attempts to discover the properties of the ether led to difficulties and contradictions. After such bad experiences, this is the moment to forget the ether completely and to try never to mention its name."
(The Evolution of Physics Einstein 1938)

"Even Einstein thinks you’re wrong"

No he does not. He does not say that there is no Aether, he only spoke of failure to make it real, and to give up and forget looking for it. That's like how Newton said in 1721 that a watch could never measure time precisely, or how Ernest Rutherford said in 1933 that anyone who looked for a source of power in the transformation of the atoms was talking moonshine.

"“Even Einstein thinks you’re wrong”

No he does not. "

Yes he does:

“All our attempts to make ether real failed. It revealed neither its mechanical construction nor absolute motion. Nothing remained of all the properties of the ether except that for which it was invented, i.e., its ability to transmit electromagnetic waves. Our attempts to discover the properties of the ether led to difficulties and contradictions. After such bad experiences, this is the moment to forget the ether completely and to try never to mention its name.”
(The Evolution of Physics Einstein 1938)

He does not say that it does not exist, his problem is discovering it.

It is a similar discussion as finding Dark Matter and Dark Energy.

Nope, he says it doesn't exist. That it existing caused only problems in explaining the evidence:

“All our attempts to make ether real failed. It revealed neither its mechanical construction nor absolute motion. Nothing remained of all the properties of the ether except that for which it was invented, i.e., its ability to transmit electromagnetic waves. Our attempts to discover the properties of the ether led to difficulties and contradictions. After such bad experiences, this is the moment to forget the ether completely and to try never to mention its name.”
(The Evolution of Physics Einstein 1938)

You, on the other hand, insist that evidence comes secondary to your screed.

"this is the moment to forget the ether completely and to try never to mention its name.”
(The Evolution of Physics Einstein 1938)

Pity, despite your fawning attitude to his quotes when they support your screed, you ignore adamantly anything that goes counter to your personal wishes.

In short: you're a crank nutcase.

Mind you, even in the statements of Einstein you gave shows that your compuslive lying has been 100% active all along:

"But this ether may not be thought of as endowed with the quality characteristic of ponderable media, as consisting of parts which may be tracked through time. The idea of motion may not be applied to it."

Einstein published the theory of General Relativity in 1916 and in 1920 he said:

"According to the general theory of relativity, space without ether is unthinkable"

Now who are you trying to fool, when he says in your quote:

"Our attempts to discover the properties of the ether led to difficulties and contradictions."

It is finding it, that is a useless task, but now we have the computer technology that he couldn't dream of back in 1938, nowadays you can create virtual mediums with CFD

http://en.wikipedia.org/wiki/Computational_fluid_dynamics

Wow it's time to grow up.

And in 1938 he said:

“All our attempts to make ether real failed. It revealed neither its mechanical construction nor absolute motion. Nothing remained"

"Nothing remained”

Now guess what the Higgs Field is, and the Higgs Boson, which should be a Scalar boson, this should mean that the Higgs-field is not like a Magnetic-field and it would be unrelated or dependent to any direction, you could than think of it as a pressure field in a water aquarium that gives mass to some particles. And we found it, or have you been Hitchhiking around the Universe these last couple of years and just got home?

"Now guess what the Higgs Field is"

A scalar field. Like height.

"and the Higgs Boson"

A force carrying particle.

Neither are the aether.

“Now guess what the Higgs Field is”
A scalar field. Like height.

You are still not getting it, it is the friction with the Higgs field that gives them mass, you can't get any friction going with 'height', perhaps today you are 'high' instead of drunk, but then again it is a substance in your blood that makes you feel this way, blurring your thoughts and vision, just like the light that we emit into space affects the Aether and makes it look as if stars are moving away us.

“and the Higgs Boson”
A force carrying particle.
Neither are the aether.

The Higgs boson itself is a vibration in the Higgs field, which can be created if enough energy is put into the field, like dropping a pebble into a pond.

"You are still not getting it,"

That's because the "it" you want me to get is nonsensical rubbish.

The Higgs Field isn't the aether.

The aether doesn't exist.

You quotemine to pretend that there is some gravitas behind you insane ramblings, when all that's behind it is just a plain old arse.

"just like the light that we emit into space affects the Aether and makes it look as if stars are moving away us."

Light doesn't interact with the higgs field, it's massless.

Moron.

"The aether doesn’t exist."

You can lead a horse to water, but you can't make it drink.

Take care.

Man who fart in church sit in own pew.

Take medication.

"Light doesn’t interact with the higgs field, it’s massless."

If so than gravity can't curve light because it's massless.

The higgs field is not the gravitational field, moron.

Once again you need to resort to using an insult to cover up your own lack of knowledge. It is a pity.

Once again you have nothing other than irrelevant "proofs".

I guess what you're saying is that the higgs are the same as gravitons because you're a moron?

You have, as always, the causal link the wrong way round.

"I guess what you’re saying is that the Higgs are the same as gravitons"

No, imho particles are temperature related. Replace mass with temperature container, and see for example the Sun as one giant cold spot, because it contains so many little cold spots. Now everything goes from warm to cold, we all look for a place in the shades to cool down. Fermions are particles that are open and Bosons are shielded off, and can carry the cold (shade) from one place to an other without losing it. Next you have to imagine the Aether as a static vibrant fluid that can transfer temperature freely but with a relative constant pressure, this would be the Higgs Field. A fermion that has mass 1 or temperature 1 and that is open, will gradually move towards the place that has temperature -1 but by doing so it will lose heat to the medium, thus the medium get's warmer around it and it's acceleration towards the cold spot will stay the same. Now imagine photons that are closed off, these particles can't mingle their temperature with the Aether (Higgs Field) so they won't change in temperature and are hardly affected by colder temperature fields (Gravity), but gravity goes in a circle around one spot (Sun) so the gradient is curved and light will be bent. Now the Aether disperses an overall a temperature of 2.725 K (CMB). And a massive particle can never move at the speed of light, because it will always lose energy because it is open, not like a photon that generates a shell and that can keep it's temperature. The relation between the temperature of the Aether and the core temperature of the particle will define the energy of a particle. And yes there might even be very small graviton-bosons that carry a little bit of coolness far into space, along with the photons, so gradually the Space around the Sun and the Milky Way is being wider and wider cooled down, affecting the photons that are approaching us. Thus the further the field around us gets to be chilled, the more energy the incoming particles relatively lose, creating Red Shift.

If you don't like it, than that's ok. There is still a lot work to do on the Virtual Aether and Organisms, to try and prove if this is possible.

"imho particles are temperature related"

They're not.

You are right, I have taken temperature here more as a popular term to describe an excitement level within the Aether and the Particles.

And, since the aether doesn't exist and temperature doesn't apply to virtual particles, it's wrong.

It wasn't your opininon anyway, you're desperately making stuff up with science words in it to pretend that you're right and that the gravitons are the same as higgs.

Your statement barely makes it to sentence, never mind hypothesis (and a long way from scientific hypothesis). And you're already wedded to the idea as shown by your use of "my opinion". Maye you're hoping that, since everyone is allowed their opinion but not their own facts, that by stating it as your opinion, nobody will be allowed to deny them.

Sorry, your hypothesis attempt is an attempt at explaining a factual event or truth about the universe.

Wow at September 2, 7:47 am

“I guess what you’re saying is that the Higgs are the same as gravitons”

Wow today at 12:32 am

"... and that the gravitons are the same as higgs."

Go take a course at comprehensive reading, or stop drinking you idiot.

I give up on you.

" you’re desperately making stuff up with science words in it to pretend that you’re right and that the gravitons are the same as higgs.".

“I guess what you’re saying is that the Higgs are the same as gravitons because you're a moron".

I can read perfectly fine.

It's you who seems to have the problem. It seems to be one at least five years old too:

http://www.ukskeptics.com/forum/showthread.php/4416-General-Relativity-…

Here is a book that you might want to read:

http://www.amazon.com/Einstein-Ether-Ludwik-Kostro/dp/0968368948

Ludwik Kostro studied physics and philosophy at the "Sapienza" University and the Gregorian University in Rome from 1963 to 1970. In 1975 he joined the University of Gdansk, until 1994 as a Lecturer and Assistant Professor in the Physics Institute and from 1994 onward as a Full Professor in the Institute of Philosophy and Sociology, of which he served as Director. He is presently Director of the Department for Logic, Methodology and Philosophy of Science at the same University. Since 1988 he has been a member of the Editorial Board of the journal Physics Essays (Ottawa, Canada).

And Nasif Hahle studied in similarly numerous universities.

However, he still can’t do simple sums:

http://scienceblogs.com/deltoid/2010/07/12/open-thread-51/

Also this idiot ignores that in 1938, Albert said:
“All our attempts to make ether real failed. It revealed neither its mechanical construction nor absolute motion. Nothing remained of all the properties of the ether except that for which it was invented, i.e., its ability to transmit electromagnetic waves. Our attempts to discover the properties of the ether led to difficulties and contradictions. After such bad experiences, this is the moment to forget the ether completely and to try never to mention its name.”

So, the paper you linked to contains his investigation in 1916.

Guess what: he found it didn't exist.

your still not getting something from nothing in any hypothesis. Now the explanation of God existing would be in fact asking one to believe something from nothing...well..firstly to provide a mother to God for example we end up in an infinite chain of mother's mother etc. In order to break the chain of infinity you would feasibly have to come to a beginning "mother" that had no preceding parent. Secondly i think a professor summed it up best by saying "God fits outside both finite and infinite" in which he expanded upon, and i won't do here because it's lengthy, my point also being, maybe just maybe there are things in the universe that we can't comprehend or explain.

"maybe just maybe there are things in the universe that we can’t comprehend or explain."

Why does that have to be God?

the problem is the word "thing". There is no such thing as a thing. At the atomic level that which is 'stuff' is made up of 99.99% empty space but is filled with abstract information, relationship, and all other abstract value concepts (none of which are measurable in our dimension). Nothing is nothing so nothing is something..."and the spirit returns to him who gave it"

Can't have a cause of the universe's creation.

Before the universe there was no spacetime.
So there was none of what we call time.
So there was no causality.

Hey trey!

The only number neither smaller nor bigger/as big as infinity are multiples of the square root of minus one.

In other words, what you're saying is that god is imaginary.

[note: technically the only numbers bigger and smaller than any real value are products or combinations of real and imaginary numbers, or even roots of negative numbers.
Note: this is based on the assumption that ∞i=/=∞

Science dumbass whose trying to learn speaking:
Is there any contradiction that in the vacuum experiment the two metal plates were pushed together while the universe is pulling outwards? Sorry if this seems flabergastingly stooopid to the better initiated.

how would proving the big bang theory help mankind and wouldnt it be a lot cheaper to just lie to us either way its been done before and im pretty sure it will happen again

until the big bang can be recreated (at full scale) then it will stay a theory. good luck

If we can create something (energy in some form or another) from nothing. Why are we not powering the world with this? I was just wondering because nothing is cheap and readily available

if you can turn nothing into something. can you turn something into nothing?

Ever seen the INSIDE of a brick, jon?

If not, the inside of a brick will remain a theory.

(NOTE: breaking a brick creates at least two new surfaces. That doesn't show you the inside of a brick)

And we can't turn nothing into something. We don't have any "nothing" left.

Something/nothing concepts probably are the convex and concave of the same curve. We seem to be describing something when we say nothing.

Maybe nothing is something like antimatter that interacts with something matter.

Also- if all physics break down prior to the Big Bang, it seems like thought/perception of such things would be impossible unless you were outside of the universe. Wasn't this godels problem?

And then you have that whole Cartesian dualism problem.

My view on this is like being born- while you can deduce your history based on evidence- you cannot really know what anything was like before you were born.

Why should the universe be different?

Maybe it was just born.

If it was- nothing in it will ever know or understand anything prior or outside it unless other universes exist and can interact with this one.

At a certain point there's an infinite regress which is probably due to faulty logic, faulty math or an inability to comprehend anything that's not bananas.

This comment thread was every bit as interesting as the blog itself, until the self righteous, argumentative, self declared, scientists of personal opinion decided to jump in (ever since it's akin to a Facebook comment thread following a Miley Cyrus joke). Thank you for the article, maybe change the title so the people that miss the point altogether don't get so butt hurt and go on and on about nothing forever.

Why does Siegel call something nothing? Inside the box is space. Space is something not nothing. Take all the molecules out and you still have space. Something, not nothing. Something is not nothing, and nothing is not something.

By Lunard Lewis (not verified) on 21 Sep 2013 #permalink

Gravity question. I don't know a lot about physics, but I was curious about something that seems like a problem to me. How is it that the gravitational pull between the moon and earth can create such massive amounts of energy to move such large amounts of water (the tides) and do this again and again every day for millions of years? It seems like you are getting something from nothing here. How can gravity create all this energy and nothing really change other than the moon just going around the earth at the same speed and distance.

No more a problem than the energy required to lift the mass of the kangaroo for one jump being more than they could possibly eat by jumping the distance from plant to plant to get it has seemingly led to the logical extinction of the kangaroo.

The energy is being borrowed from elsewhere when the tide goes up, then paid back when the tide goes down.

Lunard, that's the point of Ethan's post.

That nothing isn't nothing, it just has "nothing" (layman) in it.

Then it does have something in it.

Then nothing again.

So where did God come in? Hahaha not really! Just joshin' wiv ya! Very impressive ta

He was invented as an explanation of things.

Or, rather, evolved from the pantheistic explanations of gods for rain, trees, certain groves, waterfalls, illness, childbirth, crops, etc when people started to realise "Hey, a tiny god for each of those doesn't actually EXPLAIN anything about why they happen or not." but at this time, the idea of there being no such thing as god, no person doing things to make things happen, the next thought was "There must be a big god in charge of all these little things who set it up and makes it all keep going".

Remember: we're still tribal here. EVERY organisation had had someone in charge.

Now we're beginning to realise that God doesn't explain anything either.

Even if it is admitted that the present universe has actually come from nothing (as defined in this blog), this fails to prove that there is no god. If there is a god, then definitely there will be the hand of that god behind the creation of the universe. So non-existence of the theistic god will be established only when it is shown that the origin of the universe can be explained without invoking any kind of god. But the theistic god is also said to be all-pervading, everywhere. So, if this god does really exist, then he will be behind the vacuum created by the scientists as a background. Therefore when the scientists are saying that the vacuum is real, and that there is nothing behind that vacuum as a background, they are also assuming that the theistic god does not exist. So their starting premise is that theistic god does not exist. Staring with this premise they then proceed to show that the origin of the universe can be so explained that it will require no hand of god for any kind of intervention at any time. From this they then conclude that no creator is actually needed for the origin of the universe.
But their reasoning is circular here. This is because they have already started from the premise ‘god does not exist’. So first of all they will have to correctly establish that the vacuum created is a real vacuum, that there is no presence of god behind that vacuum as a background. For doing this, it is necessary for them to show, not by assuming that the vacuum is a real vacuum, but by some other means, that the origin of the universe can be explained without invoking any kind of god. So long as it is not done, their reasoning will remain circular here.

By Udaybhanu Chitrakar (not verified) on 20 Nov 2013 #permalink

Here I do not mean to say that there is really a god. What I mean to say is that here the procedure adopted by the scientists is not full-proof. It may be the case that there is really no god. In that case if the scientists adopt some other means instead of assuming that the vacuum is a real vacuum, then also they will be able to show that actually no god is required for explaining the origin of our present universe. But in this case we can be absolutely certain that there is no god. The present procedure fails to give us this absolute certainty about the non-existence of god.

By Udaybhanu Chitrakar (not verified) on 20 Nov 2013 #permalink

"Even if it is admitted that the present universe has actually come from nothing (as defined in this blog), this fails to prove that there is no god."

Even though this is NEVER what is claimed of science, people STILL whine about how "This doesn't prove there is no god".

The point is that if you want to claim a god exists, you need to prove that claim.

"If there is a god, then definitely there will be the hand of that god behind the creation of the universe."

Why?

Why must god go and create a universe?

Maybe it saw it already here and is watching? After all, I didn't create the TV, but I watch Corrie on it.

Lastly, the likelihood there is a god as described by ANYONE EVER is so vanishingly small you'd be better off considering the actual fact that there was a dude once who had a sleigh that flew through the sky and gave presents to kids all over the world in one night.

I.e. "There is no God" is absolutely defensible, whereas "there is a god" is not.

"So non-existence of the theistic god will be established only when it is shown that the origin of the universe can be explained without invoking any kind of god. "

That can only be established as required if you can prove that the only way universes can be created are by gods.

Scientists can show that something can come from nothing, and they can also show that our universe has also come from nothing. Then they could have stopped there. But they will not stop. Instead they will introduce god here and start an unnecessary debate by saying that no god is actually required for the origin of the universe. It is as if their first concern is not to do science, but to prove by any means that god does not exist. So what atheists are generally supposed to do is now being done by the scientists. When they will do that, we will have to point out to them that their reasoning is circular. That is all.

By Udaybhanu Chitrakar (not verified) on 21 Nov 2013 #permalink

"Scientists can show that something can come from nothing"

They have shown how this can happen and have shown that it does happen. So creating a universe from nothing can happen too.

"But they will not stop. Instead they will introduce god here and start an unnecessary debate"

No, YOU are the one whinging on here and introducing god. Nobody else is.

Scientists no more introduce god than they introduce the tooth fairy.

"by saying that no god is actually required for the origin of the universe."

Yes, when godbotherers whine about god, then this is used to say "There is no need for a god to create a universe: we CAN get a universe from nothing".

Or do you think God goes round pushing those plates together whenever the demonstration of the Casmir effect is made?

Because if not, then there's no need for god to create something from nothing.

Therefore there's no need for a god is proven by science.

That is not proving there is no god, just that, as in all other places "GODDIDIT!" claimants have had their assertions tested and been found wrong.

You know, illness, lightning, thunder, rainbows, and so on.

Is the problem that you want to be able to use the existence of the universe as proof god exists, but do not want how the universe can be created out of nothing to be used to refute your claim?

If how the universe can be created from nothing cannot be used to show the lack of a god, the existence of the universe being created from nothing cannot be used to say god exists.

Please go to your posting of November 10, 2011. There the last sentence of the last paragraph reads like this:
“How could it create something when there is complete and utter nothingness?”
So, here you are also admitting that some minimum something is required to begin with, as otherwise the outcome will be nothing again, not something. Some scientists love to define this minimum something as nothing, but they cannot compel everybody else to accept their definition. For doing this they will have to be dictators like Hitler, Mussolini, Franco, Stalin. Are these scientists having some sort of dictatorial temperament or power? If not, then other people are always free to challenge their assertion that our universe has actually originated from nothing.

By Udaybhanu Chitrakar (not verified) on 21 Nov 2013 #permalink

"So, here you are also admitting that some minimum something is required to begin with"

Nope, explicitly says right there nothing exists.

"Some scientists love to define this minimum something as nothing"

Yup, nothing is defined as nothing and when it comes to existing, nothing is a minimum: you can't get less existing than not existing.

It seems god bothereres love pretending words have no meaning.

"but they cannot compel everybody else to accept their definition. For doing this they will have to be dictators like Hitler..."

Whereas you god botherers trying to get us to accept your definition of nothing as being "God" are just like Torquemada, Hitler, Stalin (all Christians, note), Koresh, Ron L Hubbard and J Smith, right?

No, isn't saying to people a word means something merely how people communicate?

Oh, I think it is.

The problem seems to be with you godbotherers that communication is not allowed, only abject genuflection to your ideas.

"If not, then other people are always free to challenge their assertion that our universe has actually originated from nothing."

You are allowed to challenge it.

However, you can't just claim to have won because you challenge it, you have to show how you're right.

Something you completely failed to do on every turn.

God will punish you for eating so heavily from the tree of the knowledge of good and evil. Quantum Mechanics is only building another tower of Babble to come tumbling down. These mental structures trying to undue gods creation for power and profit will come to no good end. Prove all you want in the end you will crawl to the LORD. LOL Don't you just love the Christian sentiments. Makes one want to projectile vomit.

Lunacy.

Not just for the new moon any more.

“How could it create something when there is complete and utter nothingness?”
If I am not wrong, then this question expresses your bewilderment about how something can be created from complete and utter nothingness. If something cannot be created from pure and utter nothingness, then by what kind of logic can something come from pure and utter nothingness? Perhaps by some divine logic.

By Udaybhanu Chitrakar (not verified) on 28 Nov 2013 #permalink

"by what kind of logic can something come from pure and utter nothingness? Perhaps by some divine logic."

But divine logic would have to come from somewhere and you've already presupposed that:

"something cannot be created from pure and utter nothingness"

Ergo, that divine logic can't exist.

If you *are* wrong, then your conclusions are incorrect by even your assertion.

Seems like "I could be wrong" is only ever used by people who intensely refuse to believe that is the case...

Big questions:

1. Why did even existence have to exist? would not it be easier if nothing ever existed?

2. Laws of thermodynamics disagree with one great scientific discovery: universe had a beginning, how can any 'law' that the universe works by disagree with itself?

3. where did the laws of physics come from? if it is true that there were no laws of physics before the big bang, then that must also mean that there was no time, which then points to the fact that something can infact be eternal.

4. when it is impossible for the nucleus to hold together, then how does it manage to do it? answer: colossians 1:17

5. how did the bible writers manage to write over 100 scientific facts many thousand years ago, that were only discovered in the last 100-50 years? for example atoms, the expanding universe, air/wind circuits, accurate information about the deepest parts of the ocean, or how did they even know that the star cluster pleiades has tight gravity while orion has almost no gravity and is falling apart, and the accurate explanation of how the universe began and what it consists of, now confirmed: time space matter, or how did they know that there was an ice age not so long ago, also how did they even know that air has weight? many many more ...

Re:

1: Where does it say that?
2: Nope, that law doesn't preclude the universe existing.
3: They emerged as a consequence of existence. Rather like intelligence emerges as a consequence of a sufficiently complex central nervous system.
4: That doesn't answer it.
5: They didn't write 100 unknown scientific facts that only became discovered in the las 100-50 years.

Atoms: Democritus. Not in the bible.
Expanding Universe: Not in the bible.
Deepest Ocean: Nope, no cracken that lieth in the deeps.
Plieades/Orion: You have that 100% the wrong way round.
Creation: Which version? You have two. Neither agree with each other.
What it consists of: Bats are birds??? Not according to the bats...
Ice age: Where?
Air: Democritus again.

Many many more: You mean like a flat earth sitting on pillars? You mean like rainbows never existing until after a flood?

And many, many more.

ahhh.. if you want to have a serious debate i am in, but else .. dont waste my time mr wow. i'll give you an example of what i am talking about, a long one, THANK YOU if you read it to the end :) you are awesome

Time and Space:
Most people read the very first verse in the Bible, without giving it another thought. However a closer examination of this verse reveals that the author of Genesis verbalized exactly the same sophisticated principle that scientists use thousands of years later. Science expresses the universe in five terms: time, space, matter, power and motion. So does Genesis 1.

--------------------------

Atoms:
All matter is composed of atoms. Only in recent years has science discovered that everything we see is composed of things that we cannot see- tiny invisible particles called atoms, made up of electrons and protons, which are really not solids, but positive and negative charges of electricity. Whether he specifically understood it or not, the author of the books of Hebrews wrote about atomic structure, nearly 2000 years before it was discovered by scientists.

---------------------------

Gravitational Properties of Constellations:

God asked Job

“Can you bind the cluster of the Pleiades, Or loose the belt of Orion?” (Job 38:31).

“In the last century astrophysicists have discovered that the stars of Pleiades move in unison with each other, and are thus gravitationally bound. They have also discovered that the stars of Orion are free agents that are not gravitationally bound! Interestingly, the three stars that comprise Orion's belt appear to be closer together than the outer stars in the constellation, but are actually farther apart! (they appear closer together because of the 2-D plane we see them in)”. [7a] All Emphasis Added

--------------------------

By faith we understand that the worlds were prepared by the word of God, so that what is seen was not made out of things which are visible. [Hebrews 11:3] [Emphasis Added]

------------------------------

"In the beginning (time) God created (power) the Heaven (space) and the Earth (matter)... And the Spirit of God moved (motion) upon the face of the waters." [Genesis 1:1,3 … written some 3450 years ago].

-----------------------------------

An Expanding Universe:
The prophet Isaiah who lived almost 3,000 years ago stated

"It is He that . . . stretches out the heavens as a curtain, and spreads them out as a tent to dwell in." [40:22]

Scientists are beginning to understand that the universe is expanding, or stretching out. At least seven times the Scriptures clearly tell us that God stretches out the heavens like a curtain.

-----------------------------------

Air Has Weight:
The fact that air has weight was proven scientifically only about 300 years ago. The relative weights of air and water are needed for the efficient functioning of the world’s hydrologic cycle, which in turn sustains life on the earth.

When He imparted weight to the wind and meted out the waters by measure [Job 28:25] [Emphasis Added]

----------------------------------

Many, many stars...

Before people thought that there were only about 4000-5000 stars, as they could not find any more at the time, but then again, God gives incredible knowledge:

God said to Abraham 'Look now toward heaven, and count the stars if you are able to number them.' And He said to him, 'So shall your descendants be.' (Genesis 15:5)".

'As the host of heaven (stars) cannot be numbered, nor the sand of the sea measured, so will I multiply the descendants of David My servant (Jeremiah 33:22).

"Therefore from one man, ...were born as many as the stars of the sky in multitude innumerable as the sand which is by the seashore. (Hebrews 11:12)"

---------------------------------

First and Second Laws of Thermodynamics:
The 1st Law of Thermodynamics states that matter can be neither created nor destroyed, and that the amount of matter in the universe remains constant. If the First Law is correct, which every scientific measurement ever made has confirmed, then the universe could not have created itself, it must have been created in the past, no further creating must be going on, and no loss of creation is occurring. The Bible is the only religious book that correctly portrays the First Law by 1) its description in Genesis of a Creator who is no longer creating, and 2) a Creator who is "upholding all things by the word of his power (Hebrews 1:3)" .

Genesis 2:1 says (after creation):

"Thus the heavens and the earth were finished, and all the host of them."

The Hebrew word used in the above quote is the past definite tense of the verb "finish" indicating an action completed in the past. The creation was "finished" -- once and for all. That is what the First Law of Thermodynamics says. It states that neither matter nor energy can be either created or destroyed. There is no "creation" ongoing today. It is "finished" exactly as the Bible states.

The 2nd Law of Thermodynamics, which states that all systems degenerate from order to disorder, was regarded by Albert Einstein to be the premier Law in science. Again the Bible is the only religious book to accurately describe this Law:

"For the heavens will vanish away like smoke, the earth will grow old like a garment... (Isaiah 51:6)",

and also a New Testament rendering:

"You, Lord, in the beginning laid the foundation of the earth, and the heavens are the work of Your hands. They will perish, but You remain; and they will all grow old like a garment; (Hebrews 1:10-11)"

-----------------------------------------

The Hydrologic Cycle:
The Mississippi River, which is just one of thousands of rivers all over this planet, dumps over six million gallons of water per second into the Gulf of Mexico. The answer to where all that water goes lies in the hydrologic cycle, or circulation of the earth's water… something that was not fully accepted until the sixteenth and seventeenth centuries, although the earliest literature indicating an understanding of this cycle was apparently around the third or fourth century BC.. Aristotle demonstrated only a vague understanding of this process… though he recognized that rain came from clouds, he incorrectly postulated that air turned into water and vice versa.

Either way the Bible accurately portrays of this cycle 2500 years before it was widely accepted. Note the following passages:

For He draws up the drops of water, they distill rain from the mist, which the clouds pour down, they drip upon man abundantly. [Job 36:27,28]

The One who builds His upper chambers in the heavens And has founded His vaulted dome over the earth, He who calls for the waters of the sea and pours them out on the face of the earth, The LORD is His name. [Amos 9:6]

It has only recently been learned that most clouds are formed by ocean evaporation, but again the Bible had it right centuries ago:

"All the rivers run into the sea, yet the sea is not full; to the place from which the rivers come, there they return again [Ecclesiastes 1:7]".

The complex nature of how water is supported in clouds despite being heavier than air is clearly implied when God declared to Job [Emphasis Added]

"Do you know how the clouds are balanced, those wondrous works of Him who is perfect in knowledge (Job 37:16).

-------------------------------------------

Air Currents:

The wind goes toward the south, and turns around to the north; The wind whirls about continually, And comes again on its circuit. [Ecclesiastes 1:6]

The Bible describes the circulation of the atmosphere, and includes some principles of fluid dynamics.

----------------------------

Adam And Eve:
The Genesis account of creation asserts that all humans descended from the same parents, Adam and Eve. There is now considerable debate in the scientific community over recent genetic studies which indicate that all men have a common father, and all women have a common mother. In fact, the latter claim is sometimes called the Eve hypothesis. Some scientists are skeptical about these studies, and even those who are supportive would not generally accept the Genesis account; however, Bible believers should expect further research to add yet more evidence supporting these hypotheses.

-------------------------------

The End Times

Since Ernest Rutherford, who received the Nobel Prize in chemistry in 1908, is credited with splitting the atom only in 1917, it is quite impossible for people in the first century, to have known anything about the Atom Bomb. However they certainly described it well, speaking about the sky burning up and the earth itself melting. Peter certainly believed this to be true, warning people about this phenomena in a verse quite startling in its clarity.

"But the day of the Lord will come like a thief. The heavens will disappear with a roar; the elements will be destroyed by fire, and the earth and everything in it will be laid bare. Since everything will be destroyed in this way, what kind of people ought you to be? You ought to live holy and godly lives as you look forward to the day of God and speed its coming. That day will bring about the destruction of the heavens by fire, and the elements will melt in the heat." (2 Peter 3:10-12)

The prophet Zechariah also gave us a chilling message…

"Now this will be the plague with which the Lord will strike all the peoples who have gone to war against Jerusalem; their flesh will rot while they stand on their feet, and their eyes will rot in their sockets, and their tongue will rot in their mouth." [Zechariah 14:12]

While the Apostle John described the sun becoming black as sack cloth and the moon becoming like blood

And I saw when he opened the sixth seal, and there was a great earthquake; and the sun became black as sackcloth of hair, and the whole moon became as blood; [Revelation 6:12]…

which describes the phenomena that would result from massive amounts of dust and debris blown into the sky by multiple nuclear bursts. John went on to say…

"And the atmosphere was pushed apart like a scroll when it is rolled together" (Revelation 6:14).

In a nuclear explosion the atmosphere rolls back on itself It's this tremendous rush of air back into the vacuum that causes much of the destruction of a nuclear explosion. John's words in this verse are a perfect picture of an all out nuclear exchange... The whole world will be literally shaken apart!

God Bless.

If you would like to continue reading on more facts please visit: http://www.eternal-productions.org/101science.html

also I want to add to the comment #235 that the chance of the universe NOT being created by God is giving 10 monkeys one laptop each, and them punching the keyboard with their fists, and accidentaly, each of them writes one page long story, with accurate big and small letters, punctation ETC... ;)

i’ll give you an example of what i am talking about, a long one

Time and Space:

So where under that section is your proof? You've just said "People reading Genesis haven't read it PROPERLY". Begging the question.

Atoms:
All matter is composed of atoms. Only in recent years has science discovered that everything we see is composed of things that we cannot see-

WRONG.

Already told you: Democritus.

Apparently, despite your exhortation and expectation of failure for me to read your blitherings, you didn't bother reading any of mine, you had already a spiel set out you were damn well going to make, come hell or high water!

And you don't note where the bibble mentions atoms, just "Hebrews said...". Well maybe the greeks told them about it, not God.

Unless you think God (Apollo?) told the greeks first.

Air Has Weight:
The fact that air has weight was proven scientifically only about 300 years ago.

WRONG. Pre-Socratic Greece had already proof that air has weight.

The Water Thief. A water ladle. Check it out.

Gravitational Properties of Constellations:

God asked Job

“Can you bind the cluster of the Pleiades, Or loose the belt of Orion?”

God thinks that the Pleiades isn't bound and therefore cannot be the creator of it.

“In the beginning (time) God created (power) the Heaven (space) and the Earth (matter)… And the Spirit of God moved (motion) upon the face of the waters.”

Meaningless drivel. What the hell are you saying here?

God said to Abraham ‘Look now toward heaven, and count the stars if you are able to number them.’ And He said to him, ‘So shall your descendants be.’ (Genesis 15:5)”.

So you count each star. You'll fail.

Still not saying that there are more stars than are visible. Just that an ancient man who hasn't moved even to manage the number "zero", never mind "thousand" cannot count a number more than a few dozen.

This is "a lot", not a specific number of stars. And we knew there were a lot of stars 10,000 years ago.

An Expanding Universe:
The prophet Isaiah who lived almost 3,000 years ago stated

“It is He that . . . stretches out the heavens as a curtain, and spreads them out as a tent to dwell in.”

LOL!

Yeah, a tent-dwelling tribe knows that tents will expand and get bigger and yet not be moving itself...!

ROFLMAO!

If the First Law is correct, which every scientific measurement ever made has confirmed, then the universe could not have created itself,

Ahh. Seems like you never read the fucking article, you moronic bonehead.

YES IT CAN.

Air Currents:

The wind goes toward the south, and turns around to the north;

Nope, doesn't do that. Crack open a book on weather. Not a book of fiction, please.

There is now considerable debate in the scientific community over recent genetic studies which indicate that all men have a common father, and all women have a common mother.

WRONG.

The minimum pool for the mitochondrial eve's pregnancy to have resulted in us and our genetic diversity is AT LEAST 30,000.

That's rather skipped over in your "reading" of the science.

The End Times
...
However they certainly described it well, speaking about the sky burning up and the earth itself melting

The number of atom bombs we have ever produced, or could produce in a million years, all exploded together, would not cause that event.

the elements will be destroyed by fire

You can't get rid of elements with chemical processes, such as fire. So, wrong.

their flesh will rot while they stand on their feet, and their eyes will rot in their sockets, and their tongue will rot in their mouth.”

So leprosy and other necrotic diseases. Not atomic holocaust.

While the Apostle John described the sun becoming black as sack cloth and the moon becoming like blood

The moon ALREADY "becomes like blood" in a lunar eclipse and the sun won't go black, it'll become a white dwarf, but not before removing the earth from existence.

which describes the phenomena that would result from massive amounts of dust and debris blown into the sky by multiple nuclear bursts

Anything thin enough to make the moon still visible would have almost no effect on the sun's appearance. A thick cloud cover before the storm would put a thicker cloud up than all out nuclear war and that happens regularly, blotting out the sun.

“And the atmosphere was pushed apart like a scroll when it is rolled together”

So is it pushed apart or rolling together? The claim in this quote is that they are the same event, being analogised, but you wish to have them serial statements, never connected to the same point in time.

All of your apologetics is post-hoc reworking. And ignores what science has 100% proven wrong in the bible. Such as the clade of the bat. The worldwide flood fakery. Waters above the earth. Two creation myths. Two myths of humanity's creation.

And so on, and so on.

If you would like to continue reading on more facts please use the brain you think your god gave you.

Ethan, how the FK do you get quoting done under this new frigging system?

"also I want to add to the comment #235 that the chance of the universe NOT being created by God is giving 10 monkeys one laptop each"

Nope, it isn't.

"Note the following passages:

For He draws up the drops of water,"

Uh, it's water vapour, not water droplets.

You'd think the dude who had "created" the hydrological cycle would have known that.

1st of all I did read your comment to the last letter.

1. Here: “In the beginning (time) God created (power) the Heaven (space) and the Earth (matter)… And the Spirit of God moved (motion) upon the face of the waters.” [Genesis 1:1,3 … written some 3450 years ago].

2. At the time the Bible said that everything was made of atoms, there were many theories, none fully accepted at the time, there were choices, but the Bible seems to write the correct one. That is my point.

3. Here: By faith we understand that the worlds were prepared by the word of God, so that what is seen was not made out of things which are visible. [Hebrews 11:3] [Emphasis Added]

4. Air has weight was discovered .. by greeks? GREEKS had many theories covering everything both wrong and right, none of them were proven and by the way..:

source: www.nature.com

The Discovery of the Weight of the Air

A. N. MELDRUM

THE discovery, in the first half of the seventeenth century, that the air has weight is associated with things of immense importance, for instance, the invention of the barometer and the refutation of the dogma—dear to the false science and the false philosophy of the day—that “Nature abhors a vacuum”. In a new edition of the “Essais de Jean Rey”, reviewed in NATURE of July 9, an attempt is made to assign this discovery to Rey, and, so far to regard Torricelli, Galileo, Pascal, and Descartes as his disciples. Without claiming to be an authority upon Rey or upon Galileo, I would direct attention to the statement, made in “Galileo—his Life and Work”, by J. J. Fahie, that Galileo’s way of determining the specific gravity of the air was first described in his letter to Baliani dated March 12, 1613. Rey’s “Essais” was published in the year 1630.

5. The water thief? We are talking about air having weight.

6. God correctly stated that is bound, but used the sentence as a form of question, he has already bound it, but asks Job if he can do it. Anyone in 1st grade could understand this.

7. What I am saying is that the creation is clearly described here. time space matter motion power... etc.

8. 10.000 years ago we did not know that there were more stars than we could see. you do know when the telescope was discovered? not the one that can see the moon, or jupiter, but the one that can see the almost infinite number of stars 13 billion light years away? Get your facts right, thanks.

9. at the time, yes I guess the writes were in each their tents, but imagine, if you lived at the time they did, with the knowledge they had, then this is a perfect demonstration, do not expect them to say "well you know the universe is expanding at approximately "insert speed" per hour and yeah.."

10. If you feel anger starting to take over you, lift your hands over your head so that you cannot type anything mean, take one deep breath and exhale. :)

11. You really do only read atheistic propaganda? Do you not know that pilots in second world war discovered the air circuit? It is a fact now, and yes, ask anyone with knowledge about weather, they'll say it does exactly what the Bible says.

12. Why are you writing your own false facts? or whatever? do you think that a scientific study is lying?

13. I agree. But I did not say that the end time is now, either; but in a million years, do not exaggerate, please.

14. Leprosy and other? I dont think that leprosy instantly does that... no. It takes time for leprosy and "other" to do just that.

15. that event has already happened before, in 17th century in america. lasted for a full day, and without any nuclear warheads being blown.

16. have you ever seen an atomic bomb explode, on tv?
and also do you know what a scroll is? and how it is opened/closed?

everything that has wings is classified as birds. only because someone makes up new species and classifies them by "that and that" does not make it something else. it is a bird-type. or would you prefer insect?

science does not have anything against the bible nor does the bible have anything against science, if you have an example show me, but with full proof of your claim, not any theory, such as evolution.

again, for comment 239: yes it is.
again, for comment 240: remember who he is explaining it to; he is not explaining it to us, but to people that had no knowledge near us, whatsoever, would they understand it if he said vapour? no.

"again, for comment 239: yes it is."

Again, no it isn't. Casmiir effect shows that you can get something from nothing.

No god needed, unless he's pissing about pushing plates together to troll scientists for no damn reason.

Do you follow Loki? Is that it?

240 shows that the bibble is wrong: it claims DROPS OF WATER is lifted.

No, it isn't.

And what YOU Are doing is just going "I'm going to interpret what this says IN LIGHT OF WHAT HAS BEEN SHOWN TO BE TRUE, then claim this PROVES the bible was right!".

This is not valid nor even partially honest.

"1st of all I did read your comment to the last letter".

Nope you did not.

Democritus.

"1. Here: "

Is not proof of your statement "I've read every word".

"2. At the time the Bible said that everything was made of atoms, there were many theories,"

So the bibble is no more accurate than any of those others that, for example, the pre-existing Greek philosophers had.

Moreover, they had already had the idea that life grew from simpler organisms that are the origins of all life on earth and that this life started in the margins of the waters of the earth.

Since this was 3000 years ago and science says that this is possibly true only now, that proves that the biblical creation of life is wrong, correct?

After all, that's YOUR "logic" chain to prove the bible right.

Or does it only work when it comes to a conclusion you like?

"3. Here: By faith we understand that the worlds were prepared by the word of God"

By testing that faith, we discover it's a load of hokum.

"4. Air has weight was discovered .. by greeks?"

Yes.

"5. The water thief? We are talking about air having weight."

Yes, and the water thief was used to prove it by pre-socratic ancient greeks.

"6. God correctly stated that is bound"

No he didn't. The quote says NOTHING about the Pliaedes being bound. Moreover, they are not chemically bound. So it's wrong there.

"7. What I am saying is that the creation is clearly described here. time space matter motion power… etc."

You're saying that, but it doesn't follow from the things you wrote.

"8. 10.000 years ago we did not know that there were more stars than we could see."

So? The passage says that there are not more stars than can be seen. Just that you can't count them if you're the first human on the planet.

"9. at the time, yes I guess the writes were in each their tents, but imagine, if you lived at the time they did, with the knowledge they had.."

...that what they wrote wasn't about the expansion of the universe, but about how the sky was above you like the tent roof was. That the sky went down to the ground like the tent walls do, but much further away.

Like a dome of the sky.

Oh, that's right: the bibble says that, doesn't it!

"10"

ROFLMAO!!!!

"11. You really do only read atheistic propaganda? "

There's no such thing, just like there is no a-philatelist propaganda. Do you only see propaganda and "opinions" when truth and fact are mentioned?

"12. Why are you writing your own false facts?"

I'm not. Why do you think that facts you don't like must be false?

"13. But I did not say that the end time is now, either;"

Irrelevant. What's relevant is what you DO claim, not what you don't.

"14. Leprosy and other? I dont think that leprosy instantly does that"

Your quote doesn't say how long it will take. So again, irrelevant.

"15. that event has already happened before"

a) no it didn't.
b) the universe didn't end

"16. have you ever seen an atomic bomb explode, on tv?"

My dad worked on those explosions. I have a photo he's taken of one. So what was your point?

"and also do you know what a scroll is? and how it is opened/closed?"

Yes, which is why your quote is fantastically wrong. Obviously YOU don't know how they work.

"everything that has wings is classified as birds"

Insects?
Sanity towels?
Politics?

No, it was classified as that because the passage was written by people writing down the passed-on vocal stories of pre-literate stone-age tribesmen who didn't know what the fuck was going on and got it entirely wrong with regards to bats.

Something that if the message had been from god, would not have been so catastrophically wrong.

"science does not have anything against the bible nor does the bible have anything against science"

Just like that other work of fiction: The Lord Of The Rings. Or Harry Potter. Or The Ring Saga. Or Gilgamesh. Or any other work of fiction.

But when it shows that there's no need for your god as you imagine it to be, then you have a problem with it, don't you?

"but with full proof of your claim, not any theory, such as evolution."

Ah, so you get to decide what isn't acceptable as a claim.

Why not evolution?

IT DEFINITELY HAPPENS.

And it can explain all life on earth FAR better than some designer, even a psychotically maniac one.

I am not doing anything wrong, the bible was translated from hebrew, that is hard to translate; If it is read in hebrew it makes much more sense.

Casmiir effect takes place in 'free space' and nothing is mostly defined as no space at all for free movement or even existence, therefore casmirr effect cannot be taken seriously, only a false attempt to disprove us; and I do not follow anyone called loki nor do I know who he is.
if you wish to argue even more I can just add that He created us a little bit lower than Him, I believe that He imagined this world into existence by his word / thought; and we are created in his image which means that we are got something that he has, but on a lower level for ex. you can right now create your own world in your mind and everything is able to be done in that world, so therefore he created this world in him, as it is stated. We live in him, whatever he says happens, same as you can imagine a world into existence in your mind, same he did to us and everything, but on a more advanced level.

"I am not doing anything wrong"

What meaning of the word "wrong" do you mean?

Evil? No.

Incorrect? Hell yes.

"the bible was translated from hebrew, that is hard to translate"

And that means you have the retranslation right? No, I think you need to prove that your interpretation is correct a priori, not merely by taking what YOU know and saying "It says X, but it *really* means Y, which science agrees with, therefore the Bible MUST be from God!!!!1111!!1!".

"Casmiir effect takes place in ‘free space’ "

But not CAUSED BY "free space".

Read the topic before posting your reams of elephantaine shit on the thread, or pop along to where this tripe is at least condoned:

http://scienceblogs.com/startswithabang/2012/09/23/weekend-diversion-yo…

"therefore casmirr effect cannot be taken seriously"

By edict from you, right?

"only a false attempt to disprove us"

Ah, persecution complex. Wondered when that would turn up...

"and I do not follow anyone called loki nor do I know who he is."

Ah, so how do you know the Norse Mythology is not the correct information and that you're being trolled by Loki, the trickster god of that mythos?

"I believe that He imagined this world into existence by his word / thought"

Feel free to believe that codswallop.

But believe doesn't make truth.

Pop along to an opinion blog or faith blog with your tripe and leave science to the science blogs.

Do I come to YOUR place of worship and berate the pastor with "Bullshit! Rubbish! HOGWASH!!!"?

No?

Then be at least humanly polite and do the same courtesy of others here.

I shall sacrifice a black cockerel to Satan for you.

you need to involve other false gods, casmir effect requires free space in order to work. that did not exist before creation.

I came here for an intelligent debate but you need to use big letters and insulting words in order to feel control; call me back when you would like a real argument; don't forget that I told you about Romans 14:11 when you die.

"you need to involve other false gods"

So you admit that your god is as false as these others!

Well done!

"that did not exist before creation."

Creation by a false god? What? Or do you mean before the big bang? Well there wasn't any time until the big bang happened, so therefore no "before" to worry about.

"I came here for an intelligent debate"

Well, that's wrong for a start: no you weren't. You were here to complain that your personal identification of god must be right and every other identification must be wrong "because they're false gods!".

Greeks knew of atoms and the evolution of life long before those sections in the bibble and their gods included the Titans, Zeus, Apollo et al.

Since your only assertion for your god being the right one is the "accuracy" of the mythology of it, they beat yours, therefore your god must be the false one and the real ones are Zeus and so on.

PS we'll all die. It's a downer, but we're here for practically no time at all before we learn we've already got our return ticket punched.

Which leads many people to grasping at any old shite to pretend that it'll never end.

Im so sorry you got the wrong image of God, He is not someone sitting in the skies throwing lightning swords at us or hitting us with a hammer, like greek mythology says.

Why do you think thousands and thousands of people that were revived after death turn to Christ and make a testimony of heaven/hell? Many on any video broadcasting site like youtube, feel free to search it up.

Heard of Illuminati, satanists? Why do you think it is always the evil ones trying to destroy christianity in reality, why not the muslims or hindus?

I like most christians am not afraid to get into a debate, most christians fear that they will lose faith if they get beaten by an atheist or any other religious person but I cannot simply because I no longer believe in God, I now know that He exists, since He has shown himself in various ways several times. I can swear on what ever you want that these stories are true; One night at about 02.00 I noticed my cat was lost, the window was wide open and since it is an indoor cat, would not survive long, since it was cold outside, I think it was rainy also, many cars driving in the streets etc... Then after searching for 1.5 hours I asked God to bring it back, and strange enough I had no doubt that it would happen, at all. For me the only question was when, about 1 minute later my mother suddenly went out on the balcony and saw it on the busy traffic street. Later I asked her what made her do it she said that she just 'felt like' doing it and didn't know why. I never told her that it was God either, but all in one after asking Him, the cat was back home in about 2-3 minutes.

Before, when I felt like my faith was dropping, I was in a car, back seat, and told Him to boost my faith by turning on the radio in the car, and after saying that, someone asked the driver to turn it on and it was on seconds later. It really shook me.

I also have the example when I asked him to give me happiness since I was really angry at someone, after some 15 minutes I found myself wanting to run around the house 30 times because of this happiness, also I no longer felt any hate towards the person I was angry at. That kind of happiness is not the happiness when you receive a gift for christmas or a new playstation it felt awesome

This was my last comment to you, simply because you twist what I say, turn it around, insult, and cannot talk like a normal person. Bye.

Im so sorry you got the wrong image of God.

He's a fiction. As real as Gandalf.

Now, this thread is about quantum fluctuations and virtual particles. If you want to continue to spout religionist bullshit, please go to the thread provided for the nonscience you spout:

http://scienceblogs.com/startswithabang/2012/09/23/weekend-diversion-yo…

Whining on here about impoliteness means fuck all if you're going to ignore the hosts' wishes.

Because that's the height of impolite.

It is no shame to admit we simply do not understand where life came from or the universe for that matter, how it did and why.

Many good theories from both science, religions and philosophy try to answer these questions but we simply do not know for sure if the answers are true or not.

My personal opinion is that something so beyond our understanding is responsible for all of the universe and life's origination and existence. Just like humans once said and believed that no one would ever fly like the birds do and yet a few years later not only did we fly like the birds but we broke the sound barrier. My point is that the theory and materials existed for flight even though it was thought that and believed that we would never fly. We did when the time was right to understand it.

Regardless of the processes like evolution, flight,, nature, biological etc., there still has to be something that started it all and the question remains where is it, what is it, and why? to name a few.

That is why humans are constantly looking for answers because we want to know.

No one on this earth knows for sure and that is why we must all keep an open mind, always, whenever someone states they know the answers to these big questions that have been asked for centuries.

By Dr Hans Schwantz (not verified) on 13 Dec 2013 #permalink

Sorry, "Dr", the mythology department is over in the next building.

There's shame in walking into the wrong building and then spouting bollocks.

Or at least any decent human would feel shame for it.

The problem I see is that someone is trying to explain the creation of something from nothing but doesn't use nothing as the foundation. Energy, vacuum fluctuations, etc., are not "nothing." They are Physics in motion, carrying out a detailed plan of their Creator. Also, an empty box does not explain nothingness. To have nothingness, you cannot have something that can contain other things. In other words, you cannot have the box! You cannot have the Universe that has Scientific rules -- creating the "box" that can hold things -- and still have nothing!

" Energy, vacuum fluctuations, etc., are not “nothing."

However, they can arise from nothing.

"Nothing" is also part of our reality! Using these plates reveal
the actual energy. Which simply can be described, as an energy that fills the "empty" space-time. Energy that occurs, and disappears, in and out, of our reality. If we can use this for daily life energy purposes, is yet to be seen...

By Rubin Sarmell (not verified) on 30 Jan 2014 #permalink

'space' itself, x,y,z did not exist before existence. So how can you even call that empty? Not to mention the laws of physics. This is flawed.

For example, take a box and empty it, so that all you’ve got is some totally empty space, like above.
=======
The first error is the assumption that space is empty.
It is not empty, but is Spacetime.
Time is created by the interaction of particles
https://medium.com/the-physics-arxiv-blog/d5d3dc850933

Without that energy coming from somewhere else, there can be no spacetime as we know it - no space just ABSOLUTE nothing. If it is suggested that it is Quantum Energy - where does THAT Energy come from?
Energy can not be created nor destroyed, only transformed or transported ti (another universe/s)?

If space were "nothing", it could not be bent.
We know that spacetime can and is bent by mass.
The heavier the mass, the more spacetime is bent.
When we look at galaxies spinning, we have realised that there is a lot of hidden mass.
We know quantum foam produces some kind of answer, but are still adamant that space is "empty".
E=MC^2 But if E=0 then there can be no virtual particles either because some energy would be necessary to create them.
The answer is clear; energy has always existed and created our spacetime and universe, but we do not wish to admit it.

The answer to the question"CAN YOU GET SOMETHING FROM NOTHING" posted by ethan has to be logically No! if the word "nothing" means absolutely nothing then there is nothing there to make something,but we do have something the universe including life on earth so "something" has always existed therefore something is ETERNAL there's never been a time when there was nothing.You need energy to create something so energy has always existed.This does not prove God exists but allows him to exist.He must be outside the space time continueum so to ask where did he come from is an INVALID QUESTION he is the first so he must have always been there other wise you CAN get something from nothing which is impossible as just stated.phew!!

By Logic Dictates (not verified) on 30 May 2014 #permalink

You say that there is an energy field but where does that come from?

By Aaron J. How (not verified) on 05 Jun 2014 #permalink

We live in a universe that appears to begin
Fourteen billion years ago
And if what we are told can be believed
It came from nothing with nowhere to go
By nothing I mean no thing
Not some thing we call nothing
And no thing includes everything
By which I mean every thing

And not just things, this list un-includes
All sorts of anythings that could be used
All matter and energy, just for a start
All forces and fields, and every last quark
Every boson and muon and gluon and screwon
Every photon, electron, and every neutrino
Every dimension, pretension; every physics casino
Every force and attraction every quantum fluctuation
Every black hole, and every Hawking Radiation

Just keep making lists until physics is gone
Take it all away; the music and the song
Leave a bunch of nice concepts all floating in space
Then take away space and take away time
Take away every single physicist's mind
Leave nothing to chance
Leave no thing behind

Leave a bunch of laws with nothing to rule in
A bunch of ideas with no one to think them
A bunch of theories with nothing to test
Nothing to attract or repel or connect
Nothing to be and nowhere to go
Leave no thing and no where
And start with zeee-ro

See, we dream about nothing, but we all make mistakes
Our nothing is something because we're really awake
We think of nothing as if it were actual
When the fact is that nothing can’t be factual
Since nothing is the something we're trying to think
It feels like a subtle ridiculous prank
Our thinking is something instead of a blank

If we want to believe that it all came from nothing
We try to grasp nothing
As a nothing something
So here's a solution to get us to zero
To help us to grasp how it all began
To give us a little conceptual grammar
To see this fine "naught" that preceded the Bang

Nothing is less than whatever we're thinking
It's absolute absence; complete nonexistence
Without potential or possibility
A one-sided equation of zero to infinity
It can't be even thought of at all
It can't be considered, imagined, conceived
It can't be denied, ignored, or believed
Nothing is never minus nil minus naught
Nothing is complete and utter without
"Nothing is exactly what rocks dream about"

And from this, we're assured,
Without even a blink,
That all that is and all that we think,
Suddenly happened without any Cause;
That the universe and all of it's beautiful laws
Simply happened
Without a reason or rhyme;
Every iPhone, quad-copter, chainsaw, & time
Just happened to happen
By chance or by shuffle
By what Dr Seuss would call a kerfluffle

Our eager physicists can't get nothing right
They all start with something to show us the light
They all start with something since nothing's elusive
And end up with conclusions much less than conclusive
But we have to have nothing to precede the start
And nothing's a difficult thought to impart

By Phievalon (not verified) on 10 Jul 2014 #permalink

Great article, but I guess I just don't understand how experiments that prove you can get 'something from nothing' is still valid, when they had to create the experiment and an atmosphere to get the end result, which to me still shows that something had to exist or be created first. Even with "particle-antiparticle pairs that wink in-and-out of existence" could they possibly be winking in and out from one existence to another existence? How can we know?

By Daniel Colvin (not verified) on 26 Jul 2014 #permalink

sounds a bit like a capacitor to me

By steve clough (not verified) on 31 Jul 2014 #permalink

empty space is not really empty, and I guess there is no such thing called nothingness, existence is eternity.

The quantum vacuum is a type of something. It has properties. It has energy, it fluctuates, it can cause the expansion of the universe to accelerate, it obeys the (highly non-trivial) equations of quantum field theory. We can describe it. We can calculate, predict and falsify its properties. The quantum vacuum is not nothing.

don't you people get it, there has never been nothing. there was not nothing before the universe was created. the universe and fluctuating quantam particles have always existed and that time is but a measure of how long we percieve to be in existence. there is no explanation as to what was before the universe because the universe has always existed. the big bang was probably just a huge black hole that managed to swallow the entire "real matter" universe leaving nothing but quantum particle-pairs that got lucky enough to not get sucked in because they weren't considered "real" at the point right before the big bang. and then all the real matter was spit back out in the big bang to mix with all the other particle pairs.

Experiments .. in labs .. to illustrate the existence of a Quantum Vacuum first requires .. a lab. Um .. where do I get that if it ain't there? I kinda think the notion of a divine architect makes more sense. I don't understand either notion, but since I am here and thinking and seeing and feeling and loving ... it's likely because I was planned?

By Chief Suspect (not verified) on 03 Nov 2014 #permalink

@Chief Suspect #267: And if you were NOT here, does that mean your absence was also "planned"? What if one of your father's _other_ 10,000,000 spermatozoa happened to be the successful one? Would your doppelgänger hypothetical sister then have been as equally "planned" as you?

There's a technical term you may be missing out on: "contingency." It is very easy, with complex systems, to construct a clear chain of argument for how something came to be after the fact; but very impossible, with complex systems, to _predict_ an outcome before the fact.

By Michael Kelsey (not verified) on 03 Nov 2014 #permalink

don’t you people get it, there has never been nothing. there was not nothing before the universe was created.

Oh, were you there, then? Or are you just claiming this without prior knowledge? Please let us know!

The answer to the question”CAN YOU GET SOMETHING FROM NOTHING” posted by ethan has to be logically No! if the word “nothing” means absolutely nothing

However your claim here is not logical, just proclamation.Do you have any ACTUAL logic to prove your claim of "No!"? Don't just say "It must be!".

If space were “nothing”, it could not be bent.

First, yes it could, but there'd be no way to tell.

Second, the "If" is a conditional clause for the remainder that fails to pass as truth. Space isn't nothing.

E=MC^2 But if E=0 then there can be no virtual particles either because some energy would be necessary to create them

E cannot be 0, it cannot be defined PRECISELY as a single number, but only to within certain limits.

Hence, yet again, your condition does not hold, therefore the conclusion is unsupported.

fuck teory..

nothing is nothing.. no anything in there..

experiment in this blog is anything with anything has working. be anything to construct that.

nothing = 0 ok ?.. not anything in there

Ethan, you did a great job to explain this in understandable terms. Behind all experiments, though, is intelligence. In other words, it starts with something even if only a thought. This will always be the case. Any explanation that something can come from nothing is conjectural at best. I appreciate you make that known as well. We can agree that as finite beings we are limited by time and space. A true Christian perspective is NOT ALL things have a cause. Rather, it's all FINITE things and beings have a cause. God is infinite. The finite cannot fully grasp the infinite. Consequently, we all exercise faith in something. But not blind faith. Contrary to what many might believe, the Bible does not promote blind faith, nor does it renounce it. In my search for life, faith from a true Biblical perspective makes the most sense, is rational and quite logical. After thoughtful consideration, I choose (free will) belief in God as Creator who loves you and me deeply, and I can have a personal and intimate relationship with Him. He has proven Himself to me. I respect those who choose differently. It's God's greatest gift to humanity...free will.

Peace...Gunny

Behind all experiments, though, is intelligence

Not really.

Long before anyone intelligent looked at the decay rates of radioactive elements, they were decaying.

Unless you're talking in purely tautological terms, however, i.e. "Behind every experiment we've invented, there's an intelligence: ours". But that's not really all that illuminating.

A true Christian perspective is NOT ALL things have a cause.

A true Quantum Mechanical perspective is that NOT ALL things have a cause.

Rather, it’s all FINITE things and beings have a cause.

Well, THAT may be a Christian view, however not only is it not held by many christians, it's also wrong. Nuclear decay events have no cause. Merely opportunities to happen.

God is infinite.

No god isn't.

The finite cannot fully grasp the infinite.

Ergo you cannot know if god is infinite or even exists, since in either case it may be inherently unknowable or contingently unknown but still finite.

Yet this view is not held by any xtian, excepting where the predictions don't happen.

After thoughtful consideration, I choose (free will) belief in God as Creator who loves you and me deeply,

Why? How the heck does something as big as infinity notice or even care about a tiny fragment of infinitesimally small life such as yourself?

And given the actuality of life here on earth, it definitely appears as if god really hates life.

and I can have a personal and intimate relationship with Him.

Given your earlier statement re: finite knowing infinity, this appears to be contradictory.

I respect those who choose differently.

So why didn't you respect the blog and STFU about god on a science blog?

I proffer the alternative that YOU DO NOT respect IN ANY SENSE, FORM OR ASPECT those who do not believe as you do, hence you harangue and prosetylize where you are not welcome to.

Please post the location of your local church so that some atheists can pop round and talk about how god doesn't exist and that it's merely a scam for people to gain power and wealth from others, whilst justifying the personal predilections and bigotries of the people who belong (hence there being so many TYPES of christian in the world: one for each person's personal preferences).

See how well it's liked there.

Eleatic Nothingness is the absence of all positive properties.

The arguments you dismantle are straw man arguments. The unmoved mover is just that. Also the existence of time preceding a creator is a non sequitor as well

On nothingness: http://ancienthistory.about.com/library/bl/uc_bakaoukas2b.htm

on time: http://plato.stanford.edu/entries/time/#McTArg

The existence from something from nothing means that a null state can not produce a non null state. "empty space" is not a proper definition of nothing, nor does Lawrence Krauss who wrote a book about it make it so. Nothingness is a false concept. It is the absence of being and positive properties. Empty space is actually above false images, such as unicorns... and unicorns still are not nothing.

If you can write about it, it's something. Such as the casimir effect. Empty space exists within space time as an actual force. In fact, empty space may be a false vacuum, of which their may be a lower state... this lower state still wouldn't qualify as nothing; for it exists...

Nice to see that science has created "something from nothing." I am amazed at how far reverse engineering has come in such a relatively short time.

Nothing and something must exist in a super state. If the lack of all things exists (nothing), then nothing couldn't exist (something), so then it must exist (paradox or super state). Then a new question is raised, what is the something? I believe it is the fundimental representations (fr), logic and math. Yet we exist as phisical being, so some where in the fr we can measure our observations by means of comarisson of the fr we exist within. This does not mean we see all fr, for we can only observe the fr we are apart of.

"Nothing and something must exist in a super state."

No, it must NEVER exist in a super state.

It would be bad if it ever did.

"It would be bad if it ever did."
Why is that? Is it not what we currently understand?
The admittion of something is the same as nothing as non existent, which means that it does exist. Because that is what nothing is.

"Why is that?"

Because I said it.

You know, the same methodology you used.

So it's OK.

"... the same methodology you used."
You mean circular logic?
I did say paradox or super state. Besides, if exist is all 'something' in all ways, then the 'something' could not be defined. For, whatever constructs the 'something' is it's self constructed of something else unto infinitly. Which I believe is less clear than the opinion I gave. So what paradox do you suppose existence falls in?

"“… the same methodology you used.”
You mean circular logic?"

No, I already gave the methodology that you used, dearie.

"Because I said it.

You know, the same methodology you used."

You no read English so good, paul.

"You no read English so good, paul."
I am just confused as to why you think the super state cannot exist if i am using the same logic you are.

"I am just confused as to why you think the super state cannot exist "

It doesn't exist because I said it doesn't.

"It doesn’t exist because I said it doesn’t."
Well I say it does. I think of It like a spinning coin where we exist on one side. When anyone takes a picture of the coin from any place, all that can be seen is the part we exist on. Yet the other side exists because we know its a coin.

Or, its like backface culling where the experience of existing is only drawn when the existting face is in the facing direction of observation and when the opposite facing direction nothing is available to draw as existing or observable.

I see no logical problem with the duality when viewed as the liars paradox.

"“It doesn’t exist because I said it doesn’t.”
Well I say it does. "

And I say it doesn't.

Think of it as something that can't happen. Like it's not a pizza with ham and pinapple.

Think of it as the ratio of 2 lengths, it only works if nothing exists between them. If something did exist between ratios then ratios would not exist. The existence of nothing allows logic and math to work.

No, don't think of it as the ratio of 2 lengths.

Oh if nothing exists between them, then they aren't two points.

They're one.

why? Does that logic invalidate what your saying.
i meant think of the absence of something between the ratio of 2 lengths. if nothing does not exist then the ratio could not exist because something else would be between them making the simple vastly complex in calculating a ratio. Every step in aproximating the ratio would need additional formula to explain some other undefined influence on the outcome and additional formula would be needed unto infinity to get the first aproximate step. This is the paradox of only something, which is not what we observe. Only and always something leads to infinite complexity for all things. for instance, in 1+1 = x : x could never be calculated because what makes the identity and our association of 1 could not be calculated for there is no identity in always something. Only and always something is like zeno's paradox on a rampage affecting all concepts, logic, math, and reality but observable and unavoidable.

A ratio is not independant of the values comprising it, they are directly related. a division table makes the relation apparent and calculation supports this as well.

"why? Does that logic invalidate what your saying."

what logic?

"i meant think of the absence of something between the ratio of 2 lengths"

Pronoun game: not logic, half-assed quackery.

"if nothing does not exist then the ratio could not exist "

Incorrect. If nothing does not exist, then the ratio can still exist. The ratio doesn't require nothing to exist.

"and additional formula would be needed unto infinity to get the first aproximate step"

You're nearly 3000 years late. See Xeno's paradox.

"Only and always something is like zeno’s paradox "

No it isn't.

"A ratio is not independant of the values comprising it,"

That doesn't require nothing to exist.

Even if it were true, which you haven't shown in any shape or form yet.

"No it isn’t"
Yes it is, a complete answer to any concept could not exist because that would violate only and always something. a complete answer would require infinite complexity even for simple addition because mathmatical representations would not be clear as they would require only and always something. Whole numbers would be like complex numbers but much worse because they would have to facilitate always something in both explination and calculation.

" Pronoun game: not logic, half-assed quackery."
We are mathmatical representations so "pronoun game" = valid observation for physical bodies.

I have given many examples of why nothing does exist and so far all you have said for it to not exist is "I said so".

"A ratio is not independant of the values comprising it"

"That doesn’t require nothing to exist."

it does when we accept that we exist and concepts in math are physically observable, combined with the idea that phisical reality is mathematical a physical parallel must be made.

If i could drop the mic on a forum i would.

“No it isn’t”
Yes it is

No it isn't.

It isn't.

” Pronoun game: not logic, half-assed quackery.”
We are mathmatical representations

No we aren't, therefore

pronoun game” = valid observation for physical bodies.

Is incorrect.

“That doesn’t require nothing to exist.”

it does when we accept that we exist

No it doesn't. It isn't when we accept we don't exist. It isn't when we DO accept we exist. Because it doesn't require nothing to exist for us to have a ratio.

'We are mathematical representations'

"No we aren’t"

Then what are we? Seriously, if we are not the embodiment of the mathematical representations that comprise our reality, then there is literally nothing else that can make reality or us. Thinking otherwise is equivalent to an omnipotent being that is made of something other than logic and mathematics, which is completely undefinable, and yet has an uncanny similarity to the logic and mathematics we use to describe it.

'it does when we accept that we exist'

"No it doesn’t. It isn’t when we accept we don’t exist. It isn’t when we DO accept we exist. Because it doesn’t require nothing to exist for us to have a ratio."
I miss spoke, I know that it has nothing to do with accepting/rejecting existence as a whole, It does have to do with accepting what is plainly identical.

* to understand existence.

"Then what are we?"

What the hell does that have to do with what we're not?

Nothing.

We're not mathematical representations.

"I know that it has nothing to do with accepting/rejecting existence as a whole"

Then your claim that it was supposed to support is unsupported.

"* to understand existence."

*It doesn't.

Oh, by the way you're getting further and further from any point on your original claim.

"What the hell does that have to do with what we’re not?"

I'am sorry but, you are the one saying "what we are not". I am saying we are both something and nothing.

'Then what are we?'
Moreover, it seems that you have hinged the argument for the absolute absence of nothing against the physical reality of all things. I used reality as a vessel to show nothing has a place in existence, because if it exists in real life then it exists. Like smashing particles together does not give information if there is always something between them because whatever known or unknown descriptions of the particles must interact, meaning no distance, meaning nothing is between them. It might seem like a play on words but it is not, the description of no distance is the existence of nothing. I am sure you would agree that nothing does not take up space, time, or any other attribute. Nothing is the lack of a description. So it is essential that it exists as it does. It's non existence is proof that it does exist because the lack of a description is found in more places then it is not.

And I do not mean the above in just terms of language.

"It’s non existence is proof that it does exist because the lack of a description is found in more places then it is not."

let me rephrase this.

The non existence of nothing is both necessary for the existence of nothing and the negation of description. Nothing exists in more places than it does not.

I’am sorry but, you are the one saying “what we are not”.

Yes.

And my query still stands unanswered.

Moreover, it seems that you have hinged the argument for the absolute absence of nothing against the physical reality of all things.

I haven't. You've traversed miles away from your "absolute absence of nothing" to now demand we must be a mathematical representation. That you've made up something else new and unconnected with the past reality is nothing new.

I used reality as a vessel to show nothing has a place in existence, because if it exists in real life then it exists

Except your attempt did nothing of the sort.

So try something else, because that attempt doesn't do what you want it to.

And I do not mean the above in just terms of language.

And I don't mean the above just in terms of a fluid transient existence.

"The non existence of nothing is both necessary for the existence of nothing "

No it isn't.

"and the negation of description. "

No it isn't.

"Nothing exists in more places than it does not."

No it doesn't.

Let me rephrase that.

You're wandering lonely as a cloud. That floats high o'er hill and plain.

And completely lost the plot, never to find it again.

Lets get back to the original statement:

"Something and Nothing must NEVER exist in a super state."

Oh then great teacher, show me the way.

"You’re wandering lonely as a cloud. That floats high o’er hill and plain.

And completely lost the plot, never to find it again."

And that is an example of nothing. Try it again, seriously, because you keep saying "no it doesn't". Which accounts of practicing nothing yielding as much obvious affirmation that we have given, both to an fro.

“Something and Nothing must NEVER exist in a super state.”

Ok, so we know what nothing is. Now why cannot something exist along with nothing?

"And that is an example of nothing. "

No, it isn't.

"Ok, so we know what nothing is"

Nothing in what you quoted said what nothing was.

"Now why cannot something exist along with nothing?"

Because I say so.

you keep saying “no it doesn’t”. Which accounts of practicing nothing yielding as much obvious affirmation that we have given, both to an fro.

So you appear to be getting something.

At very, very, VEEEERRRRRYYYY long last.

https://en.wikipedia.org/wiki/Hitchens%27s_razor

When you proclaimed ex nihilo that you must think of nothing and something being two superstates, you wonder what the problem is.

And now, fifty posts later, you begin to understand the problem.

I even tried to make it easy for you by showing what you were doing without being you (therefore involving your own ego in pretending that what YOU did was fine).

Oh, by the way, what is your native language? It isn't English.

But why is it so? Is it because you think or understand that something cannot exist if nothing exists? Is it because if something exists Nothing is none existant? Or is it both of thouse and you do not understand the paradox that appears in the extreams of all something or all nothing?

I Gave many examples where the absence of something is obvious. As for proof, if intuition is not enough, I do not have the recources.

My native language is computer code. The known languages are so barbaric and laced with historically dissociative meaning why bother with them, except that language is all we have to communicate, and none have yet to create a better one.

"I Gave many examples where the absence of something is obvious"

No you did not. At all. In any sense of the word.

Wordsalad and stupidity? Plenty of that.

"My native language is computer code."

No, you're a fuckwit.

Enough.

"I think you are making the grander claim saying a nothing/something superstate cannot be."

And I think you are.

Back again.

Still learnt nothing you frigging moron.

Look, get with the program and try to make an argument that indicates at least SOME brainstem activity of fuck off.

Aaaahhh, live on, assembly language .......
WRT the first cartoon somewhere at the beginning of this thread, saw one a few days ago; except the free lunch was $15. Well, there's inflation still working.

JCL: a null state can not produce a non null state.

Prove it.

Fine, you are being shit on by an elephant every moment of your life. the argument of non existence, the elephant shitting on you does not exist, and what does not exist...? there also is not a line of animals training you, you are not digesting pluto, and an existence of all things cannot exist. An existence of all things would have to exist in all places and at all times otherwise, existence is not all things. Existence is not nothing, existence is not all things, So... superstate.

And who is the fuckwit "xeno".

I should say, who shares "fuckwit" status? Wow does thats who.

Yeah, you were never here to say anything a functioning intellect would do.

I Will just say that our usage of 'something' refers to 'all things'.

if 'all things' infers, all conceptual things exist and weather or not conceptual things can exist.
then 'all things' is an oxymoron.

All things implies that an apple must exist and yet, that the apple cannot exist.

If existence is not all things and the universe is not nothing
then existence is some things and some not things.

* wording
All things implies that some apple must exist and yet, that the same apple cannot exist.

*wording
then existence is some things and not some other things.

I really don't know what you are trying to tell people.

If you want to say that God did create something from nothing, from my thinking God did create something from nothing, but understanding nothing not as a material for making something, like you make bread from flour, but as the prior status which is nothing of the something which He will create.

That means, in the status in which there was nothing of flour or anything else at all, then God brings forth something like bread, so that there is the status now in which status there is bread.

Or are you telling people that in the status in which there is literally nothing at all, not even God, then this literal nothing changes into something; so God is not needed, because nothing literally nothing can create itself, i.e., change into something by its own act?

I think you are not talking logic owing to your not understanding what is literal nothing.

I don't know what you are really driving at because you write many words, but if you have a clear and precise and concise and logical point to convey to readers, you need not write so many words.

So, tell me, what is it you are really driving at.

Or you want to tell people that there has always existed the universe? But in the always existing universe there are things which change, and for that to occur you need God to exist already and He is the agent making things change.

So, you cannot at all do anything without God being existing, so that there is always something, and that is God, the creator and operator of the universe and of everything with a beginning.

What you do when you write so many words is that you really don't have a clear precise and concise logical idea to communicate, but you want people to feel something to be sensible when it is all nonsense talk from you.

But we can still say that God created everything with a beginning, starting with the universe and time and space, etc., and He did use something to make everything, His thought -- God's thought is God Himself the substance of self-existence.

By Marius Dejess (not verified) on 06 Jul 2015 #permalink

"I really don’t know what you are trying to tell people."

The truth. The truth is that you can get "something" from "nothing". And that things don't HAVE to have a cause, just the possibility.

If you're trying to tell everyone God really does exist, prove it.

Prove YOUR idea fits better than any of the thousands of other gods out there and that it fits BETTER than the nonexistence of god does.

Why should I or anyone else believe you rather than anyone else?

@ #331 Marius.
I would suggest you take note of the 'comments policy' pertaining to this blog. I'm sure there are other sites for you to expose your beliefs on. Read the question again, then ask yourself if you answered it directly, and to the best of your ability. Does it contain the proof of your claims? If not, then try again, by all means, relating and with respect to the 'policy'.

WOW again as before with the following:
If you’re trying to tell everyone God really does exist, prove it.

It's another demented question, same as the one which reads: If God does exist, why doesn't He create a rock or a boulder which He Himself cannot move?
That is one of the most cowardly questions to be asking. As I've stated somewhere else, the burden of proof is on, YOU, the scientists, to prove that God is Not.
He is the ONE who created that which you see. From the beginning, He brought EVERYTHING He created into the open, and which is still at your disposal to study and experiment with if you wish.
We all live INSIDE EVERYTHING He created, we are part of it ALL, and you still ask for more, and even also ask ludicrous, deranged questions?
The only reason you resort to such tactics is precisely because you CAN"T disprove His existence. Come on, why don't you try to prove He doesn't exist, huh?

Better yet, why don't you just wish the universe away with one of your scientific commands? Why don't you say and do the opposite of what He said and did? He said: Let there be Light, and so there was Light. So, why don't you say, let there be darkness and see if you are obeyed as He was? Why don't you try to accomplish that with all the knowledge you have of physics, of the theories, mathematical equations, and so on?
Those ridiculous questions are only a cop out on your part. That's the stubborn scientists way of twisting the, challenge, God set forth before them, so they could try to disprove His existence, but to the present, they are the ones who have always failed, and will continue to fail, to where even more of them will finally come to the conclusion that, it is utterly, IMPOSSIBLE, to disprove GOD!!! Just KNOWING, and humbly, and shamelessly acknowledging that impossibility, is...WISDOM!!!

So, to any of you who also have asked the same ignorant question of, why God doesn't create a rock He Himself can't move, the answer is very simple:
God doesn’t have to even consider, much less comply with such mindless requests, because of the fact that some of you do that for Him already, and that, on a continual basis. The proof for that is already inside your heads, because whenever someone shows you, or presents EVIDENCE of something, you instantly, and at any cost, refuse to acknowledge the truth of what you see, You simply only enjoy drowning yourselves with skepticism and denial about almost anything, or everything, at times even if it is scientific, and so with that type of mentality, which lacks so much Spiritual Consciousness and profoundness, it literally seems as if you have replaced your brains with those rocks, or boulders, to where it is absolutely IMPOSSIBLE for anyone, or anything, to move you from the position(s) where you stand on something, even though evidence shows otherwise, or the contrary of that which you so frivolously believe. You remain steadfast, just like a colossal slab of concrete or steel which can’t be easily moved. Your heads are heavily packed with those kind of rocks or boulders you ask God to create, so you do it all for Him my friends.
Just stop sounding so silly, and refrain yourselves (those who do that) from asking such stupid questions, but instead, continue, heedlessly, and recklessly, trying to prove, “THE FATHER” is not. Again, that burden is on YOU!!! Without Spiritual profoundness in your thoughts, you will never succeed, but will always only fail. You will, ALWAYS, miss the mark…which is what SIN means…
I have never understood what people like you fear so much, having to do with an intelligent Supreme being/Spirit existing? What is it you fear so much boys and girls? He would never put you in shackles. He would still permit you to continue entertaining yourselves with your work as scientists. Remember, you have free will, and that is what He gave us from the beginning so we wouldn't feel restricted in doing what we will, and which of course also includes your denying of His existence. But, the way the hatred is expressed, of even just, the idea, of the existence of such a Superior Being, is so pronounced in so many of you, that it really seems more like an immense FEAR deep inside the soul of your being, that you have to deny the Great Intelligence in a prideful, grandiose manner, (like I know it all, and I don't need anyone superior to me) simply to hide that FEAR! At least that's how some of you come across and sound like with your remarks or statements. So what if there is a God? He's a GREAT Compadre you know, and greatly to be admired!!! His amusement park is much more fun to play and enjoy ourselves in, than any other man-made amusement park. Can't you at least acknowledge that?

Or, Do you rather only continue putting more rocks in your heads, to where they are finally, shoved, or SLAMMED down to the ground, by...GRAVITY?

Fuck off you retard.

ALL HAIL EROS!

Sorry for making you explode again Puppy WOW WOWIE...the ATOMIC SHIT BOMB...CACABOOM!!!!
I guess it's because you obviously can't take the challenge to try to disprove THE CREATOR of all that is, so you have to let it all out with your CACABOOMING tantrums every time you can't have it your way, huh, Puppy loveless? Stop being so angry, and stop wasting a lot of shit already WOWIE WOW WOW, and just continue entertaining yourself by playing the Universe Game, which was set up by, THE ONE,...while you still have time, OK? Just LOVE, LOVE, LOVE...cause LOVE is all you need. Wow, Wow......Wow Wow Wow!

This didn't really answer the question at all. You used examples of preexisting things. And if particles can just pop into an empty place wouldn't that suggest that they were just somewhere else? I think you are just trying to redefine nothing. That or you don't really understand the concept of "nothing".

Yes, because we don't have any examples of a universe not existing yet.

Got any gods about you? No? Then BY YOUR ESTIMATE, you have nothing either.

Hell, show ONE example of god creating something that was witnessed.

@ rimmer #334 (slip of the finger) :)
Sounds more like you have a fear of NOT believing - off-topic again - as usual.

PJ, your and WOWIE"S (TASB) question was already answered by on of the most famous scientists, Alberto, and he says:

“Everything in Life is Vibration”

The law of nature that states everything has a vibration, and that everything is made up of atoms. These atoms are in a constant state of motion, and depending on the speed of these atoms, things appear as a solid, liquid, or gas. Sound is also a vibration and so are thoughts. Everything that manifests itself in your life is there because it matches the vibration from your thoughts.

As I've stated before, and as many others also have, and that is, that, WE ARE ALL ONE, and one with THE CREATOR, therefore every NOW moment, (which is the real and only TRUTH), is constantly being created by US, and also therefore is being witnessed by US! That right there is the "proof", or evidence you keep asking for, and it is being done in a continual basis, but can even be said...Perpetually, because that process will continue even after WE are no more in this earth to continue witnessing it personally, but who knows, maybe we will be able to continue witnessing it with our spiritual consciousness? But as I've stated before, it doesn't matter what evidence is presented to people like you, who call yourselves scientists, you have to make up excuses and CREATE new questions only because you refuse to accept what is TRUTH!

So, is your own papi Alberto E's statement nonsense? Is that not good enough for you capricious, spinning ones? You prefer creating more unmovable rocks like yourselves by indoctrinating innocent, gullible children to your believes...of non believe in a Superior intelligence who is "THE ONE" who makes it possible for Alberto's statement to be true?

He (Alberto) also says to YOU spinners:
Just because we don't understand something, it doesn't mean it isn't true.

Edgar Cayce:

Vibration is that same energy, same power, ye call God.
Know that Life itself, to be sure, is the Creative Force or God, yet its manifestations in man are electrical or vibratory.

ALL HAIL GOD...who thru a man's mind, made Eros to be...

Well....I think the possibility of having an encounter with an alien species is far greater than having an encounter with GOD!! ...Just saying.

rioma9#343 You do know that people created GOD to explain things that they couldn't explain, right? Like god of thunder, god of rain, etc. But all those things were a result of "science" that was proven later.
So what you believe as GOD is nothing more than your reason to explain something you cannot explain. But wait for some time (maybe some decades, or centuries..I don't know)
science will explain everything, you GOD FREAK!!

PJ, your and WOWIE”S (TASB) question was already answered by on of the most famous scientists, Alberto, and he says:

“Everything in Life is Vibration”

So god is a vibrator? Explains why women call it god occasionally when it's used.

Vibration is that same energy, same power, ye call God.

No, we call it "vibration". You know, nothing supernatural about it. No creation of the universe or peeking into bedrooms and occasionally into minds. No perversion, mass murders, hypocrisy, worship demands. No religion.

In short, NONE of the attributes others use when describing "god", just the attributes of "vibration".

There is something now.
What causes the Casimir effect is waves of energy being excluded from the space between the 2 plates. Because there is something now, and the Casimir effect exists only (as far as we know and can ever experimentally test) when there is something (at least energy waves of differing wavelengths), the Casimir effect cannot be used to prove that something comes from nothing.
The Casimir effect shows only that there is energy which exists in a vacuum which we cannot measure.
Quantum physicists say this is "nothing" - but then demonstrate it is something, but rather than say, "well, there is something now because there always has been something", they say, "something came from nothing" (which they have not proven. Why?

By Dana Davis (not verified) on 25 Jan 2016 #permalink

Because one is using a different meaning of the word "Nothing".

E.g.

Your mum: "Dana! What are you doing?"
You: "Nothing."

Would you think your mum was taking the piss if she said "What? Not even breathing? How about standing?"

?

Casmir shows you can get something (a force) from nothing (the lack of energy that is the zero point energy of empty space), which is a volume that has nothing in it.

Quantum fluctuation shows how something doesn't turn up for long enough to be anything at all, but still turns up for long enough to be more than its absence.

If my mum asked what I was doing and I replied "nothing" I would be correct only within a particular range of values. But I would be incorrect when considering all potential ranges of values. It is a colloquial use of the word and not a formal use.
So, quantum physicists are using the term "nothing' colloquially rather than technically? Even if the claim is that the energy in the system remains zero during quantum fluctuations of the type which led to the formation of the universe, that claim demands that there is some dimension (smaller than the dimension of the system as a whole) in which the energy is not zero for a time that is not zero - or else there are no waves nor virtual particles nor universe.
I still have difficulty understanding why the reluctance on the part of quantum physicists to use more technically correct terminology. There are better words to use to express the zero point energy in the cosmos than "nothing" such as nullity or null-space....though I guess a book called "The Universe from Null-space" would likely not sell as many copies.

By Dana Davis (not verified) on 26 Jan 2016 #permalink

"It is a colloquial use of the word and not a formal use."

Ergo different meanings of the same word.

So what is the problem?

"that claim demands that there is some dimension (smaller than the dimension of the system as a whole) in which the energy is not zero for a time that is not zero"

No, it doesn't.

"I still have difficulty understanding why the reluctance on the part of quantum physicists to use more technically correct terminology."

If it's YOU defining "correct terminology" then the reason for their reluctance is clear and obviously correct.

Proofs of the big bang are still in the scientific literature, not the local bookstore.

I appreciate your responses, though it seems my initial question goes unanswered.
"Nothing" does not have "different meanings". There may be different parameters applied to it which define the limits of what "thing" is being said to not exist, but the context of the use of the word "nothing" helps to establish those limits.
When my mum asks what I am doing and I answer "Nothing", it is because, within context, I assume she means "What activity are you engaged in above and beyond basic life functions?". If she then said, to my answer of "Nothing" said, "Not even breathing?", I would understand the limits were different than my original assumption (or that she was being argumentative) and could respond, "Oh, I'm breathing, filtering, digesting, and thinking about my wonderful mum who is so considerate of my whole well being.", as a way of answering within the new limits established by the context.
When quantum physicists say that you can get something from nothing they have established the limits of "nothing" as being the non-existence of anything. As opposed to a "zero net energy state"...for which the statement "something from a zero energy state" would be more accurate (just not as sexy).
What quantum theory does establish is that zero energy is not the same as the non-existence of energy - yet physicists' claim of "something from nothing" argues that they are the same.
I am also not sure what you mean by "YOU defining 'correct terminology' ... their reluctance is clear and obviously correct". I am not defining "nothing". It has a definition. I'm asking why quantum physicists who otherwise try to use precision in communications are using the term "nothing" in such an imprecise manner?
As to the statement about the concept of the universe resulting from quantum fluctuations demands that there be "some dimension smaller than the dimensions of the system as a whole for a time larger than zero" - this seems to be an accurate description of the Inflation Theory of the origin of the universe. Without "unevenness" in the system, there is no fluctuation (unless physicists are going to redefine system or fluctuation as well). Unevenness means that different dimensions of the system have different properties...of course Lawrence Krauss and Steven Hawking could both be wrong about this, but I'm not arguing with them. Apparently you find this doubtful.

By Dana Davis (not verified) on 27 Jan 2016 #permalink

"I appreciate your responses, though it seems my initial question goes unanswered.
“Nothing” does not have “different meanings”."

Previously on the Dana Channel:

It is a colloquial use of the word and not a formal use.

Really?

"I am not defining “nothing”."

When you claim they're not using the "technically correct terminology", what the hell are you doing if it's not defined???

"Colloquial" means informal as opposed to formal. The definition of the word does not change, just the sense in which the word is to be understood in the context it is being used. When someone asks me what I have in my empty box and I say "Nothing", I would be right if I meant there was not anything they could see with their naked eye, but I would be wrong if I was going to use the empty box to demonstrate the Ideal Gas Law. Science generally tries to avoid colloquialisms, especially in situations where the colloquialism misstates the information being presented.
It seems that quantum physicists are trying to convey fairly technical information and I just don't understand the reluctance to be precise in the words they use, especially since the word they choose to use is incorrect in the sense in which it is presented.
The statement "something from nothing" juxtaposes the two. If it is not a contrast in meanings of the words "something" and "nothing" then it more rationally should be stated "something from something". By contrasting the two, it demands that nothing cannot be something, and therefore the definition demanded by the statement is the absolute absence of anything. However, that is not what quantum physics has postulated and certainly not what can be demonstrated.

By Dana Davis (not verified) on 27 Jan 2016 #permalink

"“Colloquial” means informal as opposed to formal. "

Ergo more than one meaning.

Okay...I think I finally get it.
When quantum physicists use the word "nothing" the word does not mean "the absence of anything" the word "nothing" as they are using it means "something".
Sort of like the magician showing us an empty hat and saying "See, nothing in the hat". Though anyone who gives it any thought knows that what he is really saying is "Nothing in the hat that is evident to you." And so when the bunny appears, the illusion is complete - a bunny came from nothing.
Odd thing is that the audience knows the bunny was really there all the time and it seems to be the quantum physicists who are insisting it wasn't.

By Dana Davis (not verified) on 28 Jan 2016 #permalink

"When quantum physicists use the word “nothing” the word does not mean “the absence of anything” the word “nothing” as they are using it means “something”."

No, you don't get it.

When they do it rigorously, they use mathematics, not english words.

As an additional problem for you, you concede that what a word means depends on context, but give none for when you want to claim scientists say nothing is something. Or the other way round. You don't really say what your issue is, if it even exists, other than you don't know what's going on.

My contention is that the meaning of the word does not change, the parameters of the "thing" which there is none of when the word "nothing" is used is context driven.
In the statement "something from nothing" defines the context. Nothing is contrasted with nothing, because the statement presumes a difference between something and nothing. So, people who read popular literature or science articles, they assume the meaning is once there was not anything and then there was something - but that does not appear to be what quantum physicists mean.
My issue is that it seems to be misleading. I don't understand why the words used are more precise. It seems odd that quantum physicists are so fixated on saying quantum theory predicts you can get something from nothing.

By Dana Davis (not verified) on 28 Jan 2016 #permalink

"My contention is that the meaning of the word does not change"

Your contention is wrong. And since your bitter distrust is based on that contention that you yourself created, you really have nobody to blame but you for your worries.

It isn't my, or anyone else's, problem, you need to sort if out yourself.

I'm "bitter". What in the world gives you that idea? I have been entirely civil, just asked a question, then responded with a rational defense of my argument when you objected to it.
You appear to be the one who is carrying some emotional baggage about the use of the word "nothing"...which in the end I guess answers my original question about why quantum physicists use the word "nothing" when they clearly mean "something".
I appreciate your help in getting my answer.
It seems that you are the one who has a problem

By Dana Davis (not verified) on 29 Jan 2016 #permalink

Wow. Even goldfish have longer memory spans.

"bitter distrust"

You only had to remember until after the very next word.

Wasn't it Guth that said that the Universe was the ultimate free lunch? I don't really think that 'nothing' in the real sense exists in our universe because everything that is proclaimed to be 'nothing' always contains something. Even a vacuum is something and since the Casimir Effect we know it really is 'something'. Even the empty space in the center of the largest intergalactic voids are 'something' because it continually increases as the universe expands. And if nothing is 'nothing' then expansion would not be one of its properties. Nothing as no properties.
The Universe probably came from nothing and the probability that a universe would appear. However, I don't think a human can grasp an understanding of real 'nothing'. At least I can't.

But before space and time began? Was *that* nothing? It's not the *same* nothing as hard and absolute vacuum of space. Then again, it's only colloquially equated. The theories are quite different.

It would appear that the concept of spontaneous creation would allow for the universe to overwrite itself. This would allow for the universe to continually exist
by recreating its existence. While the concept of creating such an existence from nothing is intriguing, it would be more likely that such a concept of spontaneous
creation would be born from something rather than nothing. Therefore I view this concept as supporting the birth of the universe rather than the creation of the universe.

By Stephen J. Bauer (not verified) on 06 Feb 2016 #permalink

Something, existence, is defined by virtue of SpaceTime. Ergo, something is a product of Space and Time. As we know, the special theory of relativity allows us to view Space and Time by varying degrees; i.e., Time can slow down, Space can become unimaginely dense. Consequently, our universal existence allows for a concept of SpaceTime that exists via the dynamic environment of its varying dimensional conditions. These varying dimensional conditions would propose a universe that is motivated via its evolution from ‘minimum Space’ and ‘maximum Time’ to ‘maximum Space’ and ‘minimum Time’. The early universe was defined by its condition in ‘maximum Time’ and ‘minimum Space’. A continuum rich in being without any real atomic mass, potential energy and associated matter formed an electromagnetic universe. Upon the legacy of electromagnetic connections that defined one being in and of the universe, gravitational forces formed among the discrete mass densities of colliding energies. An evolution born of its decelerated state of ‘maximum Time’ and ‘minimum Space’ was now accelerating toward its termination of ‘minimum Time’ and ‘maximum Space’.

Nothing on the other hand, could only be realized outside of these dimsntional SpaceTime limitations that allow something to exist. Perhaps a dimensional form of Time without Space, or Space without Time. Where upon this rationalization in which the acquisition of ‘all Time’ and ‘no Space’ is the considered entry point for this universal condition, then the acquisition of ‘no Time’ and ‘all Space’ would be the considered exit point for this universal condition.

By Stephen J. Bauer (not verified) on 06 Feb 2016 #permalink

Energy is something. So is potential, which must be present for something to come into existence.

Therefore, it remains true. You can't get something from nothing.

By M.J. Cummings (not verified) on 08 Dec 2016 #permalink